Medical Expert Answers

From Student Doctor Network Wiki
Jump to navigation Jump to search

Below are some frequently asked questions that have been answered by SDN Medical experts.

If you have a question that is not answered here, please visit the SDN Confidential Consult forum to ask your question of our expert panel.

Contents

What are your views on getting informed consent of an obstetrics patient before palpating for their fetus?

I'm a first year medical student and doing a report about some issues in the healthcare system of hospitals particularly in obstetrics. I have noticed that none of the doctors bothered to get consent before palpating the patient's abdomen to monitor the fetus and i wanted your opinion, Since the patients come on a weekly basis and pretty much know the routine, is it still important to get informed consent every time or is the lack of consent not really an issue?

tantacles

Physicians may have different views on this topic, but the generally accepted view is that palpating an abdomen, because it is part of the physical exam and not an invasive procedure, does not require formal informed consent.


Underserved Communities

Hello,

Other than the people who come from low-income areas, what other groups count as underserved? Doesn't rural count as underserved because they do not typically have access to what the urban and suburban communities have?

Doesn't the population in their twenties who do not have insurance count as underserved because they typically do visit the doctor out of fear of being heavily billed?

mark-ER

It depends on what you mean by that... from a strict public health perspective, you are right. There are several medically underserved communities within the US. The ones that come to my mind, are as you say: inner city, rural w/o access to a hospital in 20-30 mile radius, Native American, immigrant communities (esp. those w/ high rates of undocumented immigrants and no insurance). Young individuals (especially men, under 35) can be theoretically considered underserved, but the underutilization of health care in that group is somewhat by choice. So the key metric of an underserved population is would pattern of utilization be significantly altered, if barriers to access (cost/convenience) are eliminated? Would healthcare outcomes improve in parallel?

So that's the public health perspective, but that != public policy perspective. From a policy perspective, be it reimbursement of student loans to those who serve in underserved areas, or where public/government grants are distributed, only some of the groups apply. Do a quick google search on this and you can go down a 'rabbit hole' quickly.


How to prepare for NBME? Short on time 3 weeks!

Hi,

I'm a Caribbean med student. I need some help how to prepare for the NBME CBSE. This is my last chance to give it and I need to score 70 or higher. My first NBME was equal to step 1 score 187. After that I scheduled for a second time but I was not able to go and my school counted as second chance. So I have only 1 chance more in 3 weeks.

I have Pathoma 6 month subscription, UWorld, and First Aid.

Is there anything I can to pass this time. Otherwise I will have repeat semester at the island.

Also what can I use for Microbiology n Immunology?

I need help please I am freaking out.

MusicDOc124

For Micro you can use sketchy micro from sketchy medical. That seems to be relatively standard, and it is what I used as well. With that said, don't go into resource overload. That can be detrimental.

I recommend looking into a single, more comprehensive program. I know people who have used DIT for instance, which worked great for them. 3 weeks is a crunch, but I know people who used it for 2-4 weeks and were fine. This would ensure you're covering all topics needed to be covered in a single program instead of spreading yourself thin across many resources and maybe missing information. Programs like that or boards bootcamp (meant for DO students in particular) cover all of the required topics/tested topics. They are expensive though.

I also recommend it because when I mention the 2-4 weeks above, that was for time until actual boards. If you were to start now, it may be only 3 weeks until the NBME CBSE, but youd have more time beyond that to prep for boards.


Post Residency Interview Thank you email

I interviewed at one of my top Internal Medicine programs last week, and made the mistake of sending 1 email addressed to the faculty to all of the email addresses sent via the program coordinator post interview. The program used the multiple mini interview model, and each of the 4 interviews were based on our answer to a different question. This made it difficult to have a unique connection or conversation point to address in different emails to specific interviewers. I heard after the fact from a friend that this is a huge mistake, and I am fearful that I will not be ranked or ranked much lower because of this. My question is, should I send individual thank you cards to just the PD and associate PD? I do not want to be seen as in-genuine for my previous email, however I do not want to seem pushy by initiating further post interview communication.

mark-ER

Hmm, interesting & somewhat difficult situation. A bit of a self-inflicted wound, as you realize. There are probably multiple takes/opinions on this, but if I were you, I would be to let the sleeping dogs lie. See how your other interviews go and if YOU really liked that specific residency and want to rank them highly, then wait a couple of weeks (till mid-January or so) and then email the program director/associate program director and let them know.


I have score above a 230's on both step 1 and 2, but submitted my application mid-october. I have applied to IM residency and have only received 2 interviews as of now (early Nov). Should I be worried?

Since I applied Mid October, when should I hear more responses? I feel like I should be competitive enough for the programs I applied to.

mark-ER

No reason to worry yet, if this is still the case in 2 weeks, you should start to (gently) inquire. It is also a bit different if you applied to 40+ vs <20 programs and if you are (or not) a FMG verus US citizen FMG vs. US medical school, Us resident, basically in that order, from shouldn't worry at all, to start to worry in 1-2 weeks. Hope that helps to calibrate.


Signed contract at the 11th hour, better job came along at the 12th. How to bow out?

I recently graduated from residency and have been interviewing for a real job. Looking in a tight market, so offers were not plentiful. I interviewed at hospital A, a frankly toxic environment with physicians who seemed unhappy in a dumpy part of town. Shortly after being offered the job, Hospital B contacted me, asking I come for an interview. Hospital B is large, well-equipped, with providers who seem happy. Hospital B took their sweet time getting me in, and in the mean time the deadline passed to sign with hospital A. Hewing to the "bird in the hand" idiom, I signed their contract. There's a non-compete clause with a 10 mile radius, and hospital B happens to be exactly 9 miles away from A. Now Hospital B is offering me a better job, and wants me to sign their visit contract. I'm wondering how best to navigate this. I presume the contract is in force regardless of whether I've accepted payment or done work for B? Would A even care if I jumped ship for B if I was never on board to begin with? I should also mention that per the contract, the hospital is entitled to all proceeds from any work done in violation of the non-compete clause.

mark-ER

No good way to do it. I would not worry too much about the non-compete; you have not even started, and it is not always easily enforceable. They will have to sue you and even if they do, no guarantee they will win and so if you get sued (big IF), you can most likely settle. An even more likely scenario is they won't sue you at all, there will just be some ill will.

That ill will/blowback is more likely to be detrimental than anything else. Especially in a very small subspecialty and especially if you are geographically restricted (& it sounds like you are, at least you would be practicing in the same zone). You must have thick skin thru all your medical training, so passive-aggresive behavior does not really matter, but if you happen to have to switch jobs again, it could be a big red flag and sabotage your chances. So my recommendation is to think long & hard. One day impressions are not always accurate. Dig up any people who previously worked at the empolyer you are currently signed at, check the turnover in their staff, do some more due diligence. If you think the difference in pay, and professional satisfaction is enough, then yeah jump ship, suffer blowback and (maybe) a lawsuit or some kind of severance. Time heals all things & this too will pass.


Foreign residency for vascular surgery

I am a fresh graduate in Turkey. I am aspiring to be a vascular surgeon in USA. I am planning to participate in the 2020 match, and until that time I can start a cardiovascular surgery or a general surgery residency here (our system is different than US). Will it affect my chances to get into an integrated vascular surgery or a surgery program? If yes, which one they will favour more?

mark-ER

Good questions. First off, make sure you have your general residency application in order. This means as good as possible, not just above average, scores on step 1 and 2 (USLME) and probably TOEFL to show off your literacy in English. In terms of vascular surgery -- unless you are a foreign medical graduate with ties within the US (family, US-born) and strong (& I mean STRONG) advocates within the USA, such as world-reknown attendings at top-10 US medical schools with 10+ publications, you are going to have a very, very hard time getting into an integrated general surg-vascular surg residency here in the USA. It is just very competitive, with very few spots. So your goal will be to get into a general surgery residency, then apply for spots as a vascular surgery fellow.

Counterintuitively, that means you would have a better chance if you pursued advanced vascular surgery in your country. It shows dedication & you will learn some skills that will be useful to you down the line.

One other suggestion and it may be a bit 'out of left field' (unusual) -- maybe you can consider military residency? US military often considers foreign medical graduates who demonstrate competency (good USMLE scores, recommendations) and it may be easier to get into, and then to get US residency status (Green Card, citizenship). It is less competitive for US medical graduate, because there are some string attached. There are several US bases in Turkey; it may help to talk to a vascular/general surgeon stationed there. Just a suggestion.


Would a PA have a good house and car?

tantacles

A PA could very well have a good house and car depending on how the PA spends his/her money. PAs are paid employees. The following site suggests that PAs in the United States have an average salary of around $92000/year:

https://www.payscale.com/research/US/Job=Physician_Assistant_(PA)/Salary

This amount of money in many places is enough o have a mortgage and pay a car loan.


Finding a job after residency

As a doctor with a medical degree in internal medicine or general surgery. What are the chances of finding a job in an area that is not where you completed your medical school and also not in the area you completed your residency? (But still in the same country)

Thanks in advance

tantacles

In most regions, it will not be difficult to find a job in any field of medicine, though finding the right job for you is always something that is difficult. At least in the United States, in urban areas, the number of jobs tends to be more, but the pay tends to be less. In rural areas, there are fewer jobs but fewer physicians want to live in these areas, so the pay is better and jobs can be easier to find.

Ultimately, the availability of jobs is region-specific.


Are residencies more likely to be in the area of your Medical School?

When choosing a medical school, is it important to choose one in the Area (Provence for Canadians) /city you hope to complete your residency in? Will my chances of getting into lets say "Vancouver General Hospital" be increased if I'm attending the University of British columbia rather than something like McGill which is in Quebec.

Thanks in advance

tantacles

It is possible that your chances will be greater of getting a residency at the institution where you did medical school, but each residency program operates differently; some places heavily favor their own students, and others less so. Ultimately, I would ask the medical schools at which you interview where their graduates match to help make an informed decision.


Remediation x 2

I am currently an M2 student. During my M1 year, I was very depressed and failed a course. I remediated in summer and passed. I just failed another course in M2 year- it is our second module. How will this effect my residency application?

tantacles

This depends on a number of factors: How your school records remediation on your transcript, what residency you apply to, and how well you do in your other courses and on Step 1. Thus, it's hard to predict how this will impact your residency application, but if anything it will be negative. I would speak to your school's administration about how these failures are recorded so you have a better idea of what you might be asked about in interviews as well as how broadly you need to apply.


added 3rd LOR after ERAS submission on Sept 15

Hi I added by 3rd LOR today but submitted my application on Sept 15. My school told us to email each program letting them know we've added a letter. Is this necessary?

mark-ER

Not absolutely necessary, but a good habit (document, document, document). Won't hurt and if the letter is indeed excellent, it ought to help. Good luck with the application/interview process!


URGENT HELP TRANSFERRING

I’m in second year in Warsaw medical university English division and I want to transfer. I’m open to any country , any uni, any time consuming procedures as long as I can transfer into third year. Any help would be great because I’m having such a hard time coping here.

mark-ER

Your question leaves me unclear as to your situation -- are you a US citizen in FMG program? If so, what presents such an 'urgent' matter that is making it seem that the best situation is to force a transfer? Transferring institutions is very hard, often requires strong extenuating circumstances (example, spouse moving to a different city). In US medical schools there is a match/trade system and it is very hard to find a match, but this is even harder to do in foreign medical schools, even those on US curriculum.

2nd year is tough for everyone, your first option should be to grin & bear it. Get as good of a USMLE score as possible, and then you are off to 3rd year medical clerkships, many often away from your current medical institution and whatever 'toxic' environment you might be facing. If your social/academic situation is untenable, I would indeed explore a transfer, as long as you maintain good (preferably excellent) academic standing. Your first choice should probably be another English-curriculum medical school in Poland (I have heard good things about Krakow and Poznan), second choice is Carribean schools with strong ties to the USA (there are only ~5-7 that are worth the steep tuition and the risk of not matching). Again, either way it's an uphill struggle to transfer... sleep on it, figuratively, for at least 2 weeks. Don't burn any bridges.


Applying to residency: past LOR

Hello, I am a US allo student applying in a noncompetitive specialty. I took a leave of absence early in 1st year due to a previously undiagnosed mental health problem. There were no academic issues and in fact, my reason for the LOA was to prevent failures or remediations and keep my academic record clean. During my LOA, I was not hospitalized, there were no legal problems, and I responded very well to outpatient therapy.

I have very much put this episode behind me and the next 3+ years have been fine: no more LOAs, no professionalism concerns, step scores are well above average for this specialty, grades/class rank average, some ECs/research. I also have two LORs from a very competitive residency in the same specialty I am applying to. Other than the LOA, the school administration has said that my application is strong.

Given these particular circumstances, what impact will this have of residency applications? I applied broadly to 40+ community and academic programs. I am scared because I took the LOA to avoid damaging my medical school record, but now it looks like the LOA itself may have irreparably damaged my record.

mark-ER

First of, good on you for taking charge of your health. That should be your priority, you cannot help and heal others, if you cannot keep yourself healthy and balanced. You definitely took the right route -- better to take a 6mo LOA than suffer failures/remediations and have to account for that. Yes, you are right having a 6mo LOA (basically a blank period) is not an asset, but as long as you are honest and forthright about the circumstances, it will not hurt you much at all, particularly among the less competitive specialties. In fact, for family medicine and psychiatry, it might actually be an asset (!) So do not worry and re-adjudicate the issue, you made the right choice, you are doing your best in ERAS and let the cards fall where they may.


applying for FM residency

In ERAS, some of the FM places are listed as "categorical" and some are listed as "Family Medicine" What's the difference?

mark-ER

I am not 100% sure (others more closely tied to family med residency can chime in), but my understanding from being involved in transitional year program recruiting, the 'categorical' program means a 3-year family medicine, terminal (no further subspecialization, no dual boarding). Other non-categorical programs may include a dual residency in emergency medicine, a combined fellowship in OB, pain medicine, palliative care and the like. I also know there are transitional year internships that then slot into either internal medicine or family medicine. If you are not sure, you can always check in with the family medicine residency director at your institution.


how many preliminary neurology programs should i apply to?

mark-ER

Depends on your stats (particularly USMLE step 1) and if you have any major red flags (academic dishonesty, failing any of the USMLE steps, failing clerkship)... if you are a standard candidate, middle-of-the-pack, most people do 15-20 programs. Red flags you can do up to 40, and if you are an excellent candidate with a very good shot at your home program where you want to stay, some people can get away with 5-10 (I would not do 1, even with a handshake agreement).


should I apply to residency program twice?

I want a spot at FM residency that is both AOA and ACGME accredited. I applied AOA and got interview. Do I need to also apply ACGME? Would that give me advantage for spots, or disadvantage?

tantacles

It is absolutely appropriate to apply for both positions. However, you may want to let the program know just for your own convenience that you are applying to both; the program may be able to consolidate your interviews into one day.


Residency Interview Help!

Has anyone used consultation services or books before for residency interview practise? If so which ones would you recommend?

Mr.Smile12

I would check if your medical school student services office has any recommendations, or could possibly arrange for you to have mock interviews. There seem to be a lot of interview questions out there posted by professional organizations in each specialty.


Elective in Mount carmel OH(Hem/Onc) vs LSU shreveport ( Acting Internship/Internal medicine)

I am a 6th year international medical student (IMG) looking to do 2 months of electives in US. I have been offered 2 electives , one in mount carmel health system OH , Hem/Onc. Other is in LSU Shreveport , Acting internship / internal medicine. what are the advantages and disadvantages of each one , what do you think is better for the future(i.e Match)


Thank you!

mark-ER

For the purposes of increasing your chance to match, they are about the same. Pick the place you want to be for those couple of months and the area that interests you (do you want to do Heme/ONC in the future?). Do well, get to know people and 2 weeks before you leave ask for a letter of support (or two). Make sure you collect them before you leave, it's harder to do it retrospectively especially from out of the country.


Matching into ACGME Peds with Comlex Level 1 491, Level 2 489?

What do you think my chances are at matching into a decent ACGME Pediatrics program with a Level 1 of 491, Level 2 of 489? I have strong LORs, CV, research, passed my PE, and preclinical grades not being super amazing but in the mid Bs. I know the cutoff for a lot of programs are >500, so unsure what my chances are.

How many programs should I apply into? I will apply AOA, however, there aren't many programs.

tantacles

It is hard to say exactly what your chances are. Each program has a different cutoff. That being said, pediatrics is one of the less competitive fields to apply to. While places that have a cutoff of 500 will likely be out of your reach, there are many that may not have those strict cutoffs and where you may get interviews.


Failed CS, have not taken CK, do I apply sept 15th or wait till I get my CS score.

I recently received my CS score and failed the ICE portion. I was in the midst of studying for CK but switched over to CS once I found out I failed. I am hoping to sit for it again and get my score back by October 12 or late October. I feel like a lost puppy after finding out I failed. Is it better to send my ERAS application when I have all my scores in and hopefully a pass on CS or is it better to send my application with my CK score and a failed attempt. I am a US-IMG, applying for psych and family.

mark-ER

Difficult position (talk to multiple people for advice, please). My first impression would be to study and practice with a friend/professional simulated patient so you are 100%+ certain you pass CS. October 15th is not too far, but CS is typically not an exam that requires enormous amounts of study time. That being said, it does have quite a few "gatcha" moments, like making sure you introduce yourself, wash your hands, etc. that could lead to a fail. In an ideal world, Ithink you can can study for both CS and CK (they are somewhat complementary). Pass CS and then hit CK hard (take CK in mid-late November), submit ERAS mid-late October as soon as you find out you pass CS, then update the application with CK score. That's my advice, but again talk to others.


Difference of Canadian and American MCAT

I won't be writing the Mcat until three years from now, but can I write my Mcat in Canada and get into a US medical school? Are Mcat's different between Canada and US?

Mr.Smile12

I am unaware of any difference in the MCAT administered in the US vs. in Canada. (The Dental Admissions Test though is different.)


Does one's amount of undergraduate loans have any effect whatsoever on medical school admissions outcome?

Trying to find out if it makes sense to avoid, as much as possible, using federal subsidized loans available for junior/senior undergrad years. Although it makes financial sense for our family to do this, we want to be sure that higher undergraduate debt does not impact admissions decisions for medical schools. (The amount we are considering borrowing is approximately half the maximum amount of undergrad subsidized loan amount available.)

Mr.Smile12

The AAMC has information regarding financial aid and paying for medical school at https://students-residents.aamc.org/financial-aid/article/first-fact-sheets/ . That said, you should talk with a financial aid officer at various medical schools to get the real answer on how your undergraduate debt may factor into your financial aid for medical school. To answer your question, financial aid issues are not considered for any consideration for admissions.


What are my options for community IM and FM through ACGME match? With COMLEX level 1 536 and Level 2 CE 491

So I am an OMSIV at GA-PCOM. I want to pursue IM or FM since I want to practice in outpatient primary care in future. My COMLEX level 1 score is 536 and Level 2 CE is unfortunately 491. As my level 1 score being in around the average I know I am looking at community IM or FM. After receiving my Level 2 CE score which dropped 45 points, I want to ask you guys that what are my chances for ACGME community IM or FM?

tantacles

It is hard to say exactly what your chances are as this is extremely program specific; that being said, most ACGME programs are primarily familiar with Step 1 scores. The exception would be programs that also have a connected DO program or accept many DO students, and these would probably be the best programs to apply to.


ENT/Surgical Specialty Residency

I'm a 3rd year medical student who has just started rotations, with surgery underway and currently on peds.

I expected to get a higher step score based on my practice exams, but ended up with a 241. I was wondering my competitiveness applying to an ENT residency program. I honored in surgery and have done well all throughout my 2nd year in the top 10% and hoping to be an AOA member. I am currently involved in research.

I know it is becoming increasingly competitive each year, just wondering if I should try and continue on a more competitive pathway or if my step 1 score puts me out of the running.

mark-ER

You are at a bit of a disadvantage, though your other endeavors mitigate things a bit. Unfortunately (just as with grades and MCAT), there has been a score "creep" on USMLE step 1 -- 10 years ago I would have said you are fine, but now you have a bit of an uphill battle. You can still very much get in... look at 'charting outcomes' -- your USMLE score is on the left hand of the bell curve, but that still means 25-30% folks match with your score or below. As to concrete suggestions, do a surgery elective in ENT early, do an away rotation or two (audition rotations), honor your M3 coursework and apply for AOA. If you do all of the above, you should be fine but even then you might have to nicely call the administrative secretaries of the residencies you really, really want to attend and make sure you do not get screened out by a strict step1 cutoff. I know doing all of the above is easier said than done, so even if those things do not happen you should still try for ENT, but might have to cast a wider net (apply to most if not all programs, and general surgery as a back door in). Good luck.

Edit: I think my answer was a bit too pessimistic. To put it another way, as an 'average' applicant with a USMLE score of 240 (per 'charting outcomes for US allopathic students) you have about a 90% chance of matching. And you seem to be an above-average candidate. Put the somewhat disappointing step 1 score behind you, and do what you can with the things you can control.


Should I do an SMP

Hello,

I am writing on this forum to seek advice on a predicament that I am presently facing. I have been admitted into a special master’s program at a University that will remain nameless; however, I am currently ruminating as to whether or not it would be a prudent decision on my part to defer my admission by one year. My plight is that I have not yet taken the MCAT, and I will eventually be required to do so to successfully matriculate into Medical School. I have two options of two different paths that I could undertake, and I was hoping that someone on this forum would be able to offer me advice on what the best course of action would be.

Option 1: I defer my admission by one year, in which case I focus on attaining the highest possible score on my MCAT examination, as well as accumulating research/clinical/volunteer experience in the interim period. If I were to do so, my application would be slightly stronger than if I were not to do so.

Option 2: If I do not defer my admission by one year, and take courses as prescribed by the curriculum of the special master’s program, then I will have to forgo taking the MCAT exam, as studying for both the MCAT and medical school courses will hinder my performance on both. I will, however, have revisit the issue of taking the MCAT immediately one month after completing the special master’s program, which will effectively give me only one month to study for the MCAT.

Regardless of which ever option I chose to take, the timeline by which I hope to apply to medical school will not change. I were to defer my admission, I would be applying to medical school for the 2020 class, while if I chose not to defer my admission, I would still be applying to medical school for the 2020 class; the only difference would be that I would be taking the MCAT now rather than later (June 2018), which may be better as the material may still be fresh in my head, and I fear that after having taking FIRST YEAR MEDICAL COURSES alongside first year medical school students, I may forget much of what I have learned for the MCAT. Furthermore, I would only have one month to prepare for the MCAT exam, as it is imperative that I take my MCAT exam in June to be eligible for the 2020 cycle. While if I were to defer my admission, I would be commencing the program without the worry of preparing for the MCAT, as I will have already cleared my MCAT exam, and I would hopefully be able to matriculate into medical school the same year that I complete the SMP program.

On the other hand, I have heard stories of a few people who have taken their MCAT after completing the program, but my guess is that these people have taken the MCAT in the past, and the period of validity of their scores has expired.

Although it may seem as if I answered my own question in elucidating my story, I hope that someone would be able to offer invaluable insight on the best possible decision, as I would like to make the most prudent decision.

tantacles

It is hard to say what you should do without more information. If your GPA is excellent, and you do well on your MCAT, there is little productive role for one of these masters' degree programs, and you may as well get work experience in a lab rather than paying for a degree which will hinder your ability to well on the MCAT and would not provide much beyond that; many of these programs do not provide degrees that help you with further employment.

On the other hand, if your GPA is sub-par, what you might want to do is take the MCAT prior to matriculating and apply while you are in the program. This would be a way to mitigate a poor GPA and possibly get a guaranteed interview.

either way, I would suggest that you do your best on the MCAT, and take the path that allows that.


I think I forgot to shake hands with my patients in Step 2 cs, though I introduced my self, told good morning and asked if they were comfortable; how would it affect my scores?

Is shaking hands mandatory in Step 2 CS exams? Do they cut your points for not shaking hands even if you introduced well by telling who you are, what you are going to do, wished them good morning and maintained an eye contact. I forgot to shake hands at my encounters and its freaking me out. Any suggestions will be highly appreciated. Thank you.

mark-ER

From what I read, it is a minor "ding" not a deal-breaker or automatic fail (versus forgetting to wash your hands). Take a deep breath... can relate, USMLE is stressful.


I have bachelors degree in civil engineering and now wants to join a med school

Hi, I have done my Bachelors in Civil Engineering in India and is now intending to join a med school in US. i am a permanent resident of US. I have already taken my courses in physics, org chem, gen chem and math in my high school as well as for my bachelors. Do i need to apply for these subjects as a prerequisite again?? Can I take my MCAT only after completing all my prerequisites??

mark-ER

It sounds like you have quite a few pre-reqs under your belt, though I would visit your undergrad institution's pre-health advisers and confirm if you can use some of the older classes, and especially high school classes in lieu of pre-reqs. I know that many medical schools would not take AP courses (chem and bio) instead of , whereas requirements for math/calculus are more loose (plus, as an engineer you probably have more advanced coursework in maths under your belt). AMCAS is another good source to investigate. Either way, I would STRONGLY encourage you to refresh your memory in chem/physics, perhaps thru an online course and spend a year in a post-bacc (formal or self designed) to get the required/recommended biology, genetics and stats coursework. Good luck.


I am marrying a non-US-IMG who will be applying this cycle, when she changes status does she need her EAD by match day or by May/June ?

Hello,

I need help with an issue. I am a US citizen currently a resident planning to marry my girlfriend who is a non-US IMG applicant. We've known each other for 4 years and have been together for 3 years. I just want to know overall for programs, when she files a change of status around October, will her EAD to be eligible to work need to be ready by Match Day or at least April May before the start of residency ? I am just trying to get a gist of how most programs work. Ive been hearing some programs want it by match day and others are ok with it by June.

mark-ER

I think this is a question where you medical school office admissions, deans office or most of all the international center (many larger universities have one) may be of most help. HOWEVER, based mostly on case-scenario experiences get the process started ASAP. You may want to even get a court marriage certificate and consult an immigration lawyer yesterday (kidding/not kidding). Once you have a firm timeline from INS/naturalization service and have that in writing, stating basically she will have legal status with right to work on July 1st, you should be OK.


Is is possible/feasible, or common at all, for a doctor to practice medicine as an internist for a few years before going into an IM subspecialty (such as hematology/oncology)?

I am considering a 3-year internal medicine residency after medical school, but am concerned that finances involved with family life will make it difficult to continue into a 3-year fellowship in hematology/oncology (or any other field I may find interest in) immediately following residency. Is it possible or common for internists to practice medicine after residency and before doing a specialty fellowship? While money is not my main focus in life, or reason for becoming a physician, it is of concern to me and my family, and will ultimately be part of my decision. I would consider any type of practice as an internist, although I am not entirely familiar with what it entails beyond primary care practice.

mark-ER

Yes, what you describe is very possible. In fact, I had mentored a resident a couple years back who did exactly that -- he was a solid (if not top notch) medical student, but a very good resident and doctor. A major 'hit' against him was that he went to a small community hospital for residency and did not have any research experience. So he chose to become a flex hospitalist, got some clinical experience on a flexible schedule including nights, and worked with me days off/weekends getting some research experience and writing up papers. Got into a really good program. There is not a TON of individuals who choose this approach, but it's not a bad one & there is some serious financial benefits to it -- paying off some student loans, maintaining IBR and being able to fund/roll over your IRA and 401k/403b into ROTH. I think the ideal time post 3-year IM before hitting fellowship applications is 2-3 years.

That being said, if you can match into a top line program, and you want to/can do a competitive IM subspecialty (Heme/onc, Gastro, cards), it is probably best to do it in succession -- 3 years + 2/3 years. Once you become an attending, with attending salary, it's hard to switch back.


what are my chance of getting in med school with 3 masters in business?

I am Hispanic who has a daughter who is a doctor and she is married to a doctor I am just 63 and I have always wanted to be a doctor. I have 3 masters in business and would love to go to medical school

mark-ER

First of, talk to your daughter and in-laws. A road to becoming a doctor is a LOOOONG one. Do you have the required pre-requisite courses (biology, biochem, chemistry, physics, etc.), completed recently and at a recognized university? Have you taken MCAT? If you do not, that's at a minimum 2 years, +1 year to take MCAT and apply, + 4 years of residency, + 3 years (absolute minimum), meaning by the time you get out to practice, you would be at least 73yo. I have seen doctors work until early 80s, but by that time they are typically on the downswing, practicing part time and doing administration/research. Now to the delicate part -- age by itself is NOT a disqualifying factor, but training a doctor is a MAJOR investment (i.e. cost) to the institution that undertakes it. So, I have seen especially among foreign nationals with a medical degree abroad retrain in the US in their 40s or early 50s, but I have never seen anyone over 60 successfully do it (brief online search only yielded one person over 60 who ever enrolled, and that person had a medical background). Also that does not take into account the money factor (tuition, expenses).


Are medical ethics the same in the UK as they are in the USA

I am soon going to apply for medical school. At the university open day they suggested reading a book on medical ethics as it will help with the interviews. I have purchased ‘medical ethics for dummies’ but have come to realise that it is about medical ethics in the USA and I am UK based and will be applying for medical school in the UK. However I can not seem to find any information that is relevant to me on the wider web as they only really seem to compare medical ethics between eastern and western countries. So if anyone has any knowledge on medical ethics in both countries I would really appreciate it as if they are the same I will be able to save money by not paying the return charges.

Mr.Smile12

I'm surprised the organizers of your open house day didn't have a set of suggested books. I'm sure the British Medical Association has some resources though their own guides may be written more for professionals. It still wouldn't hurt to see what some of their suggested issues are.

That said, I think there are many similarities in general. Scope of practice, patient autonomy, confidentiality/privacy, and conflicts of interest are similar in my perspective regardless of national health system. For a list of specific topics, go to https://www.bma.org.uk/advice/employment/ethics/medical-ethics-today .


how many programs should Non-US IMGs apply to for internal medicine residency match?

Hi! I did MBBS from India and am applying for Internal Medicine residency match this year. I have seen other IMGs applying to over a hundred places during the Match season, they end up getting about 15 interviews at the end. I want to know if there is a downside of applying to so many programs? Will it be better to apply to less programs and save the money ?

According to AAMC, point of diminishing returns is the perfect number of programs one should apply to. https://students-residents.aamc.org/applying-residency/article/apply-smart-data-consider/

Unfortunately I couldn't find any data about about IMGs. Can you please help?

mark-ER

You want to look at "charting outcomes":

http://www.nrmp.org/wp-content/uploads/2016/09/Charting-Outcomes-IMGs-2016.pdf

I absolutely agree - there is a point of diminishing returns -- you can find that point using 'charting outcomes' and I think you will find it falls somewhere in the neighborhood of 45-60. There are too many unknowns about your situation (what's your USMLE step 1 score? Do you have research experience and/or clinical experience in the USA? Do you have an advanced degree from USA like MS, MPH, MBA or PhD?) to offer more specific advice. But please tailor your residency applications to institutions where you have a connection -- places you lived, have extended family or friends, places where you mentors or writers of your letters have connections or are known. If 2-3 months into the interview season you see that you are not getting the number of interviews you expected, you can always expand your list later.


Step 1 7 weeks out, starting to panic

I'm an IMG, 7 weeks out from my scheduled Step 1. I'm looking for some words of wisdom, because I am extremely nervous about screwing this test up.

I've been doing UWorld (so far, 67% on first pass), reading FA and BoardsandBeyond/Pathoma vids when necessary. I've been through FA almost twice, and have seen all the Pathoma videos twice, B&B vids once only. I've done NMBE 13, 15, and 16 so far (all scored low 200's), and will be doing the Uworld simulated 1 next week.

I'm studying simply by doing blocks (40 q's, random, timed) and then reviewing them. This is also taking my FOREVER, sometimes I can only get through 1 block a day.

I don't know if I've hit a wall or what, but I feel like I'm not absorbing anything anymore, and my scores are pretty consistent. I'm not sure what I can do to change to improve my scores. As an IMG, I know I need a decent score to have any shot at matching to anything. I'm not looking for anything magical like a 260, but I think my base goal is a 230.

Any advice?

Thanks

mark-ER

Take a deep breath -- maybe even a long weekend off. It does sound like you are running into a wall and getting close to burn-out. My best advice would be to take the practice exam on Uworld, then see what your weaknesses are and focus on those. Yes, your goal should be 230+, though if you would be happy with family medicine or psychiatry, there is a little bit of leeway there. Is there a chance you could do an observership in a US hospital? Is your medical school known to people in the US (many medical schools in Europe like Charles University or Trinity College Dublin?) It's an important test and stressful to be sure, but it is also important to keep things in perspective, this is not your one & only shot to match for residency in the US.


Need Advice

My son (20mths) was diagnosed with a rare genetic mutation (MAP2K1, CFC). He sees a panel of specialists regularly. I'm trying to learn all I can about the syndrome. I had anatomy classes 10+yrs ago (Xray Tech, quit before clinicals, chose a different career path), but that's my only medical background. I've picked up a bunch of books on various subjects (they're all part of the "for dummies" category) & was wondering which would be best to start with. I grabbed Neuroscience, Genetics, and Molecular & Cell Biology. I thought the best people to ask would be those who are taking/have taken the classes. Any insight is appreciated & other reading suggestions welcomed. Thanks!

mark-ER

Thank you for asking, and asking in a way to educate yourself vs. seeking specific medical advice (not what this forum is for) and/or searching for unreliable/disconnected tidbits of information on the net. You have the right approach. A good place to start for someone who already has at least a little bit of science/biology background (like recent AP Biology, or some college-level biology classes) would be "Human Molecular Genetics" by Strachan (get the most recent edition, things in this field are changing RAPIDLY). If you want to learn more about your child's specific condition or similar germline childhood conditions would be some reviews on NCBI bookshelf (it can get a little bit dense): https://www.ncbi.nlm.nih.gov/books/NBK1186/ or Rare/Orphaned disease group https://rarediseases.org/rare-diseases/cardiofaciocutaneous-syndrome/ which also lists some support & social activism organizations. If in the course of your reading you ever consider getting back into the medical field, please consider genetic counseling school. It is a masters-level program, with only ~20 in the country, but the need is growing exponentially, especially in cancer genetics.


skills of medical researcher

I was thinking of skills med students should gain from med school ... it seems I just study ( certainly high marks in exams is not the ultimate goal of med school) beside I am more interested in academic and research work rather than clinical and can't know what skills are requiered , how to gain them, which sites to visit or references to use , and if I want to attend research institution after graduation what kind of qualifications I must have ?

mark-ER

Hard question, and one that has bothered clinical educators/medical school administration for some time. On one hand, most medical students SHOULD develop critical thinking skills, writing, problem solving that is necessary for research (clinical and/or basic). On the other hand, others would argue you should develop those skills ideally before medical school (hard core undergrad) or perhaps after. Medical school is like an army boot camp. There is an ever expanding torrent of clinical information and skills you have to learn, which largely precludes doing the hard, critical thinking necessary to learn scientific inquiry. Add to that scientific inquiry skills are best learned at the bench, with empirical testing, one-to-one mentorship and you can see why it's been more and more difficult to squeeze it into medical training, more realm of the lab departments (maybe, and I stress maybe pathology) and history books (age of armchair/gentleman scientists). Add to that hierarchy of medicine (do not question authority), heavy grant writing involved in research and time stresses of both medicine and research and you can see how it is becoming harder and harder to gain those skills.

That being said practical advice -- there are programs out there (Duke in particular) that take advantage of the lulls of medical school to squeeze in an independent research project. If you are truly interested in research and building your career as Physician-SCIENTIST, rather than PHYSICIAN-scientist, then MD/PhD is a pathway you should strongly consider (either thru your own medical school or NIH). Lastly there are ways to get postdoc training or even ad-hoc PhD during/after residency (UCLA Star comes to mind, but there are others). Good luck. Perhaps another MD/PhD expert can chime in.


What is the difference between a family doctor doing cosmetic procedures and a plastic surgeon?

I'm a first year student interested in doing family medicine but not sure if I can see myself dealing with the usual cough and cold, stomach ache, diarrhea-type situations on a daily basis. I think it would be cool to do minor surgeries, including botox. What can and can't a family doctor do in this area? Also, why would one choose to go to a family doctor for cosmetic procedures instead of a plastic surgeon?

mark-ER

In a word -- complexity of cases. Another few words -- sensitive anatomic location (think eye, face, hand), place there is a lot of skin tension/movement, in general requires more hands-on practice and training. It is a loooong road to become a plastic surgeon, and if you want a variety of procedures, and complex cases, especially reconstructive (trauma/burn) or oncology, you will have to do plastics. Conversely, you can focus your practice to be more procedurally oriented in family practice -- just as you can focus on OB, or sports medicine, or pain medicine. Family = good deal of flexibility, both in terms of scope and autonomy, but somewhat less complexity/more referral and (cough) less prestige and compensation. Make sense?


selecing a book

what is a the best surgical book for a medical student ?a book that is not too long

Renee_MD

I recommend Case Files and UWorld Q Bank for each clinical rotation during medical school. These will really help you prepare for the shelf exam at the end of the clerkship. In addition, one book that really helped me was 'Surgical Recall'

Good luck!

Dr. Marinelli


What school should I go to???

I want to go to an accelerated med program (I.e. Sophie Davis, University of Toledo,Drexel, etc.). But I'm not sure if it's the right choice, because I'm interested in specializing and becoming a surgeon. What school or schools would you suggest I apply to that doesn't require and MCAT and possibly is a 7-8 year program ? Or do I have to go about the "normal" way? Because I want to go directly to med school after high school. Thanks

mark-ER

My strong advice is for you to explore the profession further. Shadow doctors & nurses (yes both are valuable, in different ways), maybe get involved in a research project. If you are a rising junior in high school (i.e. sophomore this year), you still have time. Otherwise, I would recommend traditional college-medical school route. Lastly, trying to avoid having to take MCAT is not a valid reason to choose to go accelerated route. You will have to take many, many difficult (& much longer) exams throughout your career. Good luck with your choices.


DO student interested in GS at Mayo (MN). Chances? Should I audition?

Hello, I am a DO/MBA student interested in CT surgery. I am trying to figure out where I should do my away rotations. I am very much intrigued by Mayo Clinic's 4+3 Thoracic track and their GS program. Do I have a realistic chance at this institution? Will an audition boost my chances?

Step 1 - 251. Publications/Abstracts/Presentations - 10+

Thanks.

mark-ER

Very solid stats, congrats. Short answer, double & triple check that Mayo actually takes DOs into their surgical program -- some programs have a strict rules, as in MDs and American grads (almost) exclusively. You can look at their last 10+ years of match list, to see where their prior matriculants came from. Even if they have not previously taken a DO, I would encourage you to very gently & respectfully reach out to the GS residency director program. Explain your interest, specifically why you are so very interested in Mayo. And yes, you should audition rotation, maybe even do two audition rotations in two different departments within surgery. Do well, get good letters and good luck with the match.


How to become a medical artist

Hey there!

I'm currently in my first year of med school. I've always had this passion for medicine as well as arts. Both these fields are quite interesting and both have their own equivalent importance.

So I looked it up and to my surprise, I found out that one could be a medical artist/ illustrator. The use of this is kind of self explanatory i.e. a medical artist creates visuals, textbooks, 3D models, 2D models, videos, photographs and much more to contribute to the healthcare system. And that's what I intend to do as of now, despite being in the first year and having a changing mind.

My question to anyone reading is this: What are the requirements for becoming a medical artist AFTER med school. Keyword "med school" after your bachelors. Please. I've referred to a few sites here and there but they weren't that much a help to me. Please do share if you do know anything about yet requirements example do you need a MD, and a degree in visual arts or something like that. I'd really appreciate it. Thank you.

mark-ER

Ha, interesting question. You can become an artist/illustrator after medical school, though I would caution you that it is an extra year (or more), while you are NOT earning a DR. salary (opportunity cost), presenting a potential red flag (undecisiveness) to residency programs, and you are accumulating interest on your loans. If you really want to pursue a career in medical illustration, perhaps finishing medical school may not even be the right choice (?? though I would think very long and hard about that one). The best advice I can give you -- perhaps explore this passion thru a gap year, between 2nd and 3rd year of medical school? The main program I know that has a strong reputation is at Hopkins (http://medicalart.johnshopkins.edu/). I also know somebody who followed a path from medical illustration to MD, though she practices medicine basically exclusively now. PM me and I can put you in touch.


Primary Apps for Medical School

Do adcoms take into account your major and school during primary apps? Or is it just a stats game with GPA and MCAT?

How in-depth do adcoms look into primary apps? Do they care about work experience/personal statements/letters of recommendations, or is that just for secondary apps?

Thanks!

Mr.Smile12

It's too hard to generalize for all admissions committees. A lot will factor in your major and school when looking at your GPA. Many will look at your descriptions of activities from primary and secondary applications, not to mention essays in both applications.


Help with choosing AI or Electives for General Surgery

Hi all,

I am a 3rd year Medical Student applying for away rotations for the upcoming year. I am interested in General Surgery for my residency match. I am currently designing my 4th year schedule to help me be the most prepared for intern year by transferring through different subspecialties and gaining experience in a wide range of different surgeries. I am also hoping that my AI will help me match at the hospital I'm doing the elective at. To that end, will accepting a Vascular Surgery AI at an institution that has an integrated vascular residency help me match at that hospital for General Surgery? Will transferring through Vascular Surgery help prepare me for PGY-1? Should I instead go for an AI at a different institution for a more core General Surgery subspecialty such as Transplant, Trauma, or Minimally Invasive instead?

Thanks for all your guys help!

mark-ER

You are overthinking this. Go in, do well, work hard, learn and enjoy the journey. No matter what surgical subspecialty you do, it matters most how well you take care of patients & get along with staff (residents, fellows, attendings), rather than any specific subarea. That being said, if you have a specific interest or want exposure to an area that is not very well represented within the hospital affiliated with your medical school, go for that. Vascular is a good choice, but so is trauma and minimally invasive.


What are my chances of getting into a direct medical program?

Hi, i am currently a junior in high school and i am interested in getting into a direct medical program, but i feel as if i dont have the chances. My SAT score is 1430, my weighted gpa is 4.12. In addition to academics, im currently shadowing a doctor, i have been doing martial arts for 8 years, i am going into a medical internship at Palomar Health in san diego, and have founded a Red Cross club at my school. Plz tell me which direct programs i should apply to (MD and DO) and my chances of getting in(average or low).

Mr.Smile12

This is probably not the forum to give you advice on specific programs to apply to, but I'm pretty sure there's a forum topic available to you. As it stands you should be able to search for such guaranteed admission programs through the Medical School Admissions Requirements resource from AAMC. You should be able to filter for Combined Degrees > BS/MD programs.


step1 preparartion

I am a recent medical graduated,i came to NYC in october,preparing in kaplan but i dunno why i don't have the confidence i know the concepts.. People all over are advising me to read study and do assessments...some advice me to do UWorld .some say don't waste time ,apply for MPH.. i dont know whether i am ready for it or not ..but i wanna be a doctor and work here and help as many people i can ...that us why i chose to be a doctor,i really need an expert advice

mark-ER

Looks like you are a FMG, so yes you have an uphill battle in front of you. Review outcomes of the match (AMA/AMCAS) for your desired specialty, specifically looking at FMG stats and that will give you a score range to aim for. The study advice you were given is solid. Review suggestions on the forums here and a study schedule on affiliated site. Remember US medical grads study for 3-6 weeks (closer to 6 these days) full time for this test and USMLE step 1 is considered the biggest single hurdle to get your choice residency, and even more so for FMGs since for many on the residence committees your medical school reputation/grades are an unknown.

You can do MPH if you think it would be useful; it can be an asset for an application (esp. in internal medicine and similar fields), give you a more solid statistical background, opportunity to improve your English/polish off your interview skills and you can also study hard for USMLE step 1 at the same time. Also remember, you need to take & pass step 2 (+ step 2CK) before residency applications.

So again, long road ahead, but many many people have done it, so you can do it too. As a bit of inspiration, in medical school I used to take a bus with an Armenian gentleman who was in his mid-late 40s and only later found out he was a PGY3 surgery resident with 2 teenage kids and switched back to medicine in his 40s.


Specialty IMG in Australia

I'm an otolaryngology resident in Tehran, Iran. What are my chances to work as an specialist IMG in Australia when I finish my residency? Is there any chance my residency will be considered as completely or partially comparable to the australian training? On averege how long will the assessment process take?

SamburgerHelper

Hi -

Thank you so much for your question. Our medical school experts don't currently have expertise in international-related questions. We would recommend asking your question in the international medical forum (https://forums.studentdoctor.net/forums/general-international-discussion.85/). The SDN international community might be able to provide some specific insights to help answer your question.

Thank you, SDN Experts Staff


Hopeful admission to medical school

Hey, I’m 24 and recently graduated with a BS in psychology. I had a 3.2 major GPA and a 3.1 cumulative, however extenuating circumstances played a role such as; long term illness and death of close family, financial issues that required me to hold 2 sometimes 3 jobs. I was set to attend a social work graduate program but realized it wasn’t for me. I have done an internship in a psychiatric hospital aswell as another mental health outpatient facility. I have readmitted as an undergrad to major in biology, to get my pre reqs, and boost my GPA. I intend on retaking some classes I did mediocre in to show I can do the work. As well as doing research with a professor. If I can boost my GPA to at least a 3.3-3.4 and do very well on the MCAT, do I stand any chance of being admitted to a medical program? I’ve read the statistics that even people with 3.0-3.1 have chances with good MCAT scores.

mark-ER

Sounds like you have a plan... stick to it & hopefully you are attending a post-bac program that's on par or better profile than your undergrad. You have an uphill climb for sure, but if you really push you can get to your goal. Keep an open mind, being MD psychiatrist is not the only path to get to do what you want to do in a greater scheme of things: consider DO, PsyD and maybe even genetic counseling (fascinating career & great job market).


residency in the EU

i'm a french lebanese 6th year medical student in lebanon(i hold the french passport) studying at the lebanese university , im trying to understand how i can apply for any residency program in any eu country ( preferably france , belgium , ireland , switzerland) and i heard that i need to make my degree equivalent to the EU countries degree , how is that done .

SamburgerHelper

Thank you so much for your question. Our medical school experts don't currently have expertise in international-related questions. We would recommend asking your question in the international medical forum (https://ift.tt/2JfySbl). The SDN international community might be able to provide some specific insights to help answer your question.

Thank you, SDN Experts Staf


How does one prepare for the Patient Note in step 2 cs? My typing speed is 41-44 wpm and I'm always stuck with an incomplete PN.

mark-ER

Your typing speed is not the issue, 30-40 wpm should be plenty. Being thorough, accurate yet succinct is key. First Aid for step 2 CS is the ultimate (and some would argue sufficient as the "only") prep book.


failed step 2 cs and residency start

Just found out I failed step 2 cs for the second time. I already matched into my top program. I registered to take it again may 14th for the results to be possibly out on june 20th. Orientation is june 15th and im suppose to start working on july 1st. My medical school also requires a passing score for graduation. Any advice on what to do? Would my program be willing for me to start late? Is there a possibility of a deferral or would I lose my residency position? Pls help, I have no idea what to do...

mark-ER

Yes, I think all the things you listed are possible. Talk to the medical school career counseling and somebody friendly within the administration (perhaps an assistant dean, or whoever wrote your dean's letter when you applied to the match). If indeed you take it again on 14th and get results on the 20th, it may really be 'water under the bridge', at least for your immediate career goal of starting a residency. But your #1, #2 and #3 priority right now is passing step 2 CS. Look over your scores, see what you could have done that caused a fail. Do anything and everything not to fail for the 3rd time.


Failed step 1 and passed on 2nd attempt. What are my chances of matching residency?

U.S. citizen. 3rd year medical student at M.D. school. School is pass/fail system with no letter grades. Receiving average of mid 70s on shelf exam. Failed step 1 with 191 first attempt. Step 1 second attempt score was 213. 1 research publication. No volunteer or leadership positions.


Want to match into internal medicine residency. Eventually want to be either hospitalist, intensivist (critical care), or cardiologist.

Definitely acknowledge that critical care and cardiology fellowship programs may not be possible at this time.

What are my chances of matching into an IM residency program?

mark-ER

A failure on step 1 is seen as a red flag by many programs and will nix your chances of an interview at vast majority of competitive/academic programs. Community programs, especially in geographic areas where you might have a strong connection (thru your background, family or a mentor) are your best bet, though consider applying broadly to both community and low-mid tier academic programs. Competitive subspecialties, especially cardiology within IM, may be quite a stretch, unless perhaps your home institution is very impressed by you. Consider family medicine applications as well (backup plan, back door into being a hospitalist).


Pursuing a residency 7 years after graduating medical school

I graduated from medical school in 2011 with excellent Step 1 and Step 2 scores. Since then I have been working as a Scientist in the the field of drug discovery and development. I would like to pursue a residency in pathology but fear that it is too late for me to return to a clinical career path. Is there a way?

mark-ER

It is definitely not too late. Your issue may be licensing, especially the long interval between residency and USMLE step 1/2 and then step 3 (which presumably you did not take yet). Re-taking USMLE step 1, especially after such a long break, is not a fate I would wish upon my worst enemy so perhaps there is ways to get around it, by petition? I don't know specifically, but I think I know someone who does. PM me and I will put you in touch.


One of my parents was diagnosed with schizophrenia shortly before I took my step 1. Should I mention overcoming this adversity on my residency application?

About 1 month before taking my step 1 exam, my mother was diagnosed with schizophrenia. It came out of nowhere and was devastating for me and my family. I moved back home to help take care of her and get her the treatment she needed (my father was out of the country on a business trip at the time). As you can imagine, this situation drastically affected my ability to study and perform, and ultimately negatively influenced my step 1 score (I scored in the upper 220s). Based on my previous academic performance, and the circumstances of the effect that my mother's diagnosis had on me, I am certain that the score is not a true reflection of my ability. I did not report this adversity to my medical school at the time because I wanted to avoid the stigma of having a first-degree relative with such a notoriously debilitating mental illness. But upon reflection, I'm thinking it is an adversity that might be worth mentioning on my CV and application to residency, especially considering that my performance on the step 1 exam is not consistent with my academic abilities. What are your thoughts on this issue? Should I mention this adversity? What are the risks of mentioning this adversity, if there are any? What are some possible implications of mentioning this adversity that I should be aware of?

Thanks so much for your time and advice.

mark-ER

I think you will get multiple different opinions/perspectives on this. Here's mine -- it depends. To me, this would be a private matter, and by bringing it up in your application you invite someone to discuss it or at least mention it. Are you ready for a discussion about the subject, in context of your career? Are you applying to a very competitive residency, where a step 1 score in the 220s would be below 25%ile? If the answer to either of these is 'no', don't don't bring it up. Your other question -- any risks of mentioning this adversity? Ideally, no but theoretically maybe. There is a reason why we have a mental health awareness month, and there is still some stigma attached to mental health issues for society in general and to a lesser degree within medical professions. *None of the above applies if you are heading into psychiatry.

Lastly, it sounds like your mother's diagnosis also really affected YOU. Consider getting some private counseling, to help you think thru and process her diagnosis for yourself.


General Surgery Application - Competitiveness and Places to Consider

Hello,

I am trying to apply for general surgery residency next year and wanted to gauge the strength of my application/what are realistic expectations for where I could land. I have been researching programs that I would like to go to, but I would still like suggestions for places I should consider.

My Information:

State school Step 1: 241 Step 2: taking in few months Clinical grades (ABC system): 4 As and 2 Bs (IM and Peds) Pre-clinical grades: all As MSPE comments: pretty happy with these, some of my best comments actually came from my B grades, which is bittersweet haha Not AOA ECs: lot of volunteering and organization leadership Research: 10+ posters, 4 non first authors, 3-4 pending first authors, 2 oral presentations Other: couples matching with someone doing EM and similarly competitiveness as me

mark-ER

You will be moderately competitive (a good shot, but not a guarantee) at many academic hospitals, just below top tier (i.e. top 5-10 hospitals), which should be the bulk of your applications and rank list. With appropriate connections and letter writers you may even have a shot at the top programs too, though couples match complicates things a bit. For smaller, community-based programs you will be very competitive. I would encourage you to expand your list to 20-50 programs, and make your lists together with your significant other, taking into account geography, personal connections and career goals (i.e. do you want to stay in academics?) This advice is coming from someone who is involved in residency selection committee for transitional year programs (surgery and medicine), so perhaps someone specifically with general surg residency background can chime in as well.


Chances of matching into integrated CT (I6) or integrated vascular surgery?

- 247 on Step1 - 2 publications - One 1st author in vascular surgery, one 3rd author in neuroscience. - 10+ presentations (poster/oral) in national & regional conferences - No demonstrated CT Surgery in research/activities - top 1/3 of class, but no honors in clerkships except IM - Relatively new medical school

tantacles

It is hard to say exactly what your chances are. They definitely are not 100%, and they will depend, ultimately, on who else applies. I would recommend looking at Charting the Match to get a better idea of your chances. Your advisers at your school may be able to help you figure out your exact chances of matching, and if there is no one who can help you, it may be wise to reach out to attendings in this field at other medical schools.


is it possible to become a neurosurgeon as a D.O

Hello,

            I have always been interested in neurosurgery  as long as I can remember and now that I'm getting closer to make this  dream a reality I wanted some advice before making  any decision on how I should approach this. Generally, most neurosurgeons are M.D.'s, however I have found myself more intrigued with the D.O. program (not due to grades, but just interested in the holistic aspect of medicine ), and was wondering if there is even a route that leads to neurosurgery.  With all this said, I do understand that my interests may change as i am going through residencies, fellowships etc., but until that day doesn't come I want to keep my goal set on neurosurgery.
Any kind of suggestions to comments would be appreciated. Thank you

Mr.Smile12

There is no reason why DO's cannot become neurosurgeons when it comes to a general overview of the medical education process. With the recent efforts to merge allopathic and osteopathic residencies, the lines are blurring and should continue to do so. But get as much mentoring as you can about the career path.


do degree in pure biology have chance to be medical doctor?

do degree in pure biology have chance to be medical doctor? i want to pursue in medicine, but somehow i end up taking degree in pure biology which take 4 years. is there a way to pursue in medicine with this degree? and how?

tantacles

Many people in the United States become physicians after doing degrees in biology. In the United States, you would complete a four year degree in any major, complete the pre-medical requisites along with any requirements the school might have. They subsequently apply to and are accepted to medical school.


Did not match into Peds =( , what do I do to strengthen my CV to increase my chances for next year's match? Thanks.

I am graduating as a US-IMG this year. I have passed my USMLE Step 1/2CK/2CS on the first try with my grades being 210, 216, PASS, respectively. I applied for the 2018 match but unfortunately did not match so I was wondering what I should do to strengthen my CV and my chances to match next year.

mark-ER

Insufficient information to give a truly informed answer to your question: how many programs did you apply to? Are you restricted in terms of geographic location? Are you doing couples match? Any serious red flags in your application (academic dishonesty, fails in any of the clerkships?) Have a long, introspective review of your application, talk to the residency director for peds at your home institution and form a solid plan of action as to what to do with your application next year and what you would be doing in the intervening year (MPH, research, medical mission)... Though it may be of little solace, you are not the first and you won't be the last person in this position. Good luck.


Post-Graduate Physician Assistant CardioThoracic Surgery Residency?

I'm a current PA student and have a very strong interest in going into Cardiothoracic surgery or Vascular Surgery. Has anyone been to a residency program for this? If so, what are your thoughts?

I have seen several options to do so, just wanting other opinions. Thanks!

SamburgerHelper

Hello,

Thank you for your question. At this time, none of our SDN Experts have expertise in this area so we will be unable to assist with your question. Please feel free to check our Clinicians [RN / NP / PA] Forum (https://forums.studentdoctor.net/forums/clinicians-rn-np-pa.113/) for assistance with answering your question.

Sincerely, SDN Experts Manager


Medical research for undergraduate in developing country like Nepal

I am a medical student final year at a respectable university in Nepal. I have intentions of applying in US. I have heard a lot about research and other requirements for better matches. How do I get involved in research here(Nepal)? I have no clue about any undergraduate being involved in any kind of research paper or anything like that over here. Would really appreciate some guidance.

SamburgerHelper

Hello,

Thank you for your question. At this time, none of our SDN Experts have expertise in international research for undergraduates so we will be unable to assist with your question. Please feel free to check our International Forums (http://forums.studentdoctor.net/forums/general-international-discussion.85/) for any additional information on this topic.

Sincerely, SDN Experts Manager


Didn't match/ telling programs

I didn't match into Derm this cycle. What's the proper way to tell prelims and Derm programs I didn't match and I am still interested in their programs.

tantacles

Dermatology programs likely already know that you did not match as you fell below the other applicants on the programs' rank lists. I do not think there is any need to let programs know you did not match; rather, I would suggest that you work on your application for next year or the year after as you will likely want to apply again if you are still interested in dermatology.


ranking of Rheumatology Fellowship

I wanted to know the ranking of Rheumatology Fellowship so that I can go to a medical school that has a connection with those programs.

tantacles

There is no widely accepted ranking for fellowships, but many private websites have rankings of these programs. I would suggest that you perform a web search for "rheumatology rankings," and this might give you insight.


Easiest state(s) to get medical license with combine 12 month residency experience

I am a US medical graduate and switched between two different residencies. For example I started off in Internal Medicine and got 6 months of credits. Then about 2 years later I got about 8 months of credit in another specialty for example Pediatrics. I did have issues with my program director at my second residency but my Chair is supportive of me and would sign off on my Medical Licensing application, which is possible with some states as it states on the letter (PD and/or Chair). While speaking to a some state medical board, they have told me it is possible to combine different residency programs as they have seen it before.

I realize I have an uphill battle given I did not finish two different residency, and I kindly ask if anyone here knows which states and/or US territory is the easiest to obtain my medical license. I am hoping to get a medical license and either practice in that state/US territory and/or work for the Federal Government. Any help would be appreciated.

God bless and i hope to hear from you soon

mark-ER

I am not sure, you may want to consult a credentialing expert at your local medical center or a medical school. But as the question has been sitting unanswered for a while, I will give it my best shot. As you have noticed, there is a LOT of variability as far as paperwork requirements, credentials and general helpfulness of the staff from state to state. I have either past or present accreditation (medical license) in 6 states, and by far the easiest in my experience had been Pennsylvania. It also doesn't hurt the fees there are quite low. Good luck.


I am a first-year medical student struggling to find research plans for the upcoming summer

I do not have any major research experience under my belt (no papers, publications). I am currently working on a minor survey study this year with some students at my school for a poster presentation. I would really ideally want to expand my research experience but I feel that every researcher i have contacted already has a full lab or is not willing to give me a starting point. I am not sure how to go about expanding this part of my studies; also my med school really is awful at providing us with research opportunities or outlets (hence why I am stressing out and on here). Any tips, advice, suggestions would be helpful! I do understand how important research is in the Match process.

Mr.Smile12

Hopefully your student services staff will be helpful, as would your supervising professor for your survey. Otherwise, many medical schools with research infrastructure will often advertise for medical student research fellowships, and some other research institutes that are not directly tied to medical schools will also publicize similar fellowships (such as the National Institutes of Health/Howard Hughes Medical Institute). You may want to see if there are still opportunities to apply for summer research fellowships through those organizations. There is also a possibility you can ask residency directors for suggestions to do research that may be relevant to your ultimate career direction.


What is the meaning of aging gametes?

tantacles

Aging gametes refers to the aging of reproductive cells, usually ova, as their quality degrades overtime and can lead to issues with nondisjunction and birth defects related to the resultant genetic problems.


USMLE: help needed!

Hey! I needed a little help in figuring out some facts regarding usmle. Currently I’m going for an externship to Chicago. I’ve not appeared for both step 1&2 . I’m an intl med. graduate. Wanted to know if I’m right giving my CS exam first. Could you guide me a little about the preparation and how to crack it with the least delay? Thanks.

tantacles

There are many study materials available for the USMLE Step exams. I would suggest that you look in the USMLE forums on SDN for advice on which materials are best to use; there are many available resources, and everyone has a different strategy. You can take the exams in any order that you wish to, though US students traditionally take Step 1 followed by Step 2 CK followed by Step 2 CS.


US IMG chances in matching in IM with a pass in the core clerkship

Hi, I got a pass in my school's internal medicine core clerkship, and I wanted to know what my shots are for actually matching in IM, otherwise I'm thinking of doing peds instead if I can't do IM. Some of my background: I am in international medical school, with a step score of 243, haven't taken step 2 or CS yet. I have gotten an honors in peds and psych, haven't completed my other clerkships yet. I did do one research project, but no pubs/posters.

Mr.Smile12

Did you ask your clerkship director about his/her thoughts about your performance? You should try to talk with your student services advisor as well about how your STEP score and letter of support regarding your clerkships in general could help set up your match application.


what are my chances of residency in internal medicine as an IMG in US

Hi, I am an international medical graduate from India , passed my medical school in the year 2013, did my MD in radiation oncology from India. I have passed the usmle step 1 with score of 231, step 2 Ck 242 and step 2 cs all in first attempt. I am currently doing observerhips in Chicago, I have completed observerships in cardiology, nephrology from elmhurst memorial hospital , illinois 8 weeks, have a 4 week radiation oncology observership at RUSH university medical centre. , 4 week observership in neuro oncology from RUSH university, chicago . I have 3 letters of recommendations from RUSH university. I am interested in shifting to medical oncology for which i want to do internal medicine. I also have 12 publications and a research thesis from india. What are my chances ? I am planning to apply for 2019 match year. I am also planning to take the step 3 before the applications.

tantacles

It is hard to say exactly what your chances are of getting an internal medicine residency. Every program has different requirements, so you may want to speak to your mentor at your current program and the programs you have rotated at to find out your chances at those specific programs.


usmle preparation

hello, please give me some methods of studyig for usmle step 1 and what study materials to start with, im a freshly graduate from medical college in Bangladesh.i am hoping to do the exam in November 2018.is the period enough to prepare?

tantacles

My personal preparation included several runs through pathoma, several runs through Uworld, and a several runs through first aid, a book that can help you prepare for step 1. I would also recommend doing practice tests to gauge your progress.

Every person has a different process, and there are many resources available in the USMLE Step 1 forums on SDN that I would suggest you peruse.


Do undergrad extracurriculars still matter once you are in medical school?

I am a 6 year BS/MD student who will be starting medical school in the fall. I'm not sure if this is the right question category, so I apologize if it is not. So my situation is a little different than the average premed student because I did not really need to have extracurriculars during undergrad due to being in a direct med program. I did pretty much nothing my first year and my second year of undergrad I have been trying to get involved as much as possible to build a resume. However, I am not sure if this is even going to be helpful for me which is what my question is about. Will undergrad research and experiences matter once I go to medical school or will nobody care? I was under the impression it could make applying to research labs in medical school easier, but I am not sure if this is true. I know residencies will look more at your time in medical school, but should I bother worrying about my undergrad resume? Examples of things I've done: Research lab assistant, peer mentoring, medical club member, women in science club member, some volunteering Things I want to start doing before the end of the semester: free clinic volunteering, present research at a symposium, get a job at a child development center (considering going into pediatrics)

tantacles

The main extracurriculars that will matter for applying to residency will be research. Other activities, while they are meaningful and may be great when applying to residency because they can be great for conversation during interviews, are unlikely to be significant. In medical school, the most important thing is to get the best grades you can and do your best during your clinical rotations.


away elective opportunity resources outside VSAS

Are there any other database resources listing away electives and sub-internships, outside of VSAS, but similar, for third year medical students trying to streamline the application and search process for the fourth year away elective rotations? Also, same question but for RS (rural health setting) electives?

tantacles

While I am not aware of any databases, it might be worthwhile to contact individual hospitals and programs to see if they offer experiences outside of VSAS. This could lead to a positive experience with a school, and some schools, perhaps even your own, might be willing to create an elective for you.


Universities offering medical sub-specialities without the need of Internship after graduation in English

I am a 6th Year MBChB student. I will be finishing my medical degree in December 2019. I will be needed to do an internship of one and a half year to pursue any specialisation in my country after completion of med school. I am not very enthusiastic about doing an internship before specialisation. I am willing to join any sub-specialities or masters programmes that would help me make a better career that do not require internship as a pre-requisite. I do not have any specifications about universities. Good recognised universities anywhere around the world that provide english courses with reasonable tuition fees or that provide scholarships is all i am looking for. Any help will be highly appreciated. Thank You In Advance

tantacles

It is my strong suggestion that you check the SDN forums for suggestions about international schools that have inexpensive English courses. I imagine the websites of these universities would be great resources as many of these institutions post their tuition publicly.


NBME 13: 232. My goal 250

Hi there!

I have gone through FA and pathoma 3 times and UW qbank twice.

- UWA1= 245 (2 month ago) - FA revision (after UWA1) - UWA2= 243 (1 month ago)

Then I decided to do 40% of USMLERx and then did one intensive pass of FA in 2 weeks. Then I did NBME13.

NBME 13= 232 (last week).

I am really disappointed now, I thought I was really doing great and ready for the test since time is crucial since I would like to apply this year.

When I revised the test I realized at least 7 of my mistakes were really misunderstood while I was reading the question. There were 26 total wrong answers.

I sat with my self to do a plan to get 245+ and I realized based on the NBME 13 result I should follow 8-week study plan as follows: - Doing the remaining of USMLE Rx (10 days) - Redoing UW (18 days) - NBME15+16 - one FA pass (10 days) - NBME 17+18 - NBME 19.

Do you find this 8-week plan reasonable? I was thinking before I can apply this year but I am afraid I can not since I have not done STEP2 CK and CS yet. Would you suggest going with a shorter plan? or stay with this plan and take my time with the steps and apply next year?

Your input is highly appreciated.

Thanks.

tantacles

Your 8 week plan seems overall reasonable as a study plan for Step 1. Keep in mind that each practice test is different and has different scoring criteria, so your score varies. USMLE world may overpredict or underpredict scores, as can NBMEs. Ultimately, it is hard to know if you will be successful in attaining your goal score as performance on Step 1 is highly individual. I would suggest, however, that you take the time you need to get the score you want, if possible, as it is impossible to take step 1 more than once if you pass.


I want to apply for match this coming season. What paperwork should i start getting ready (I am an IMG) and when does the process start?

I just finished my Step 2CS and other steps. But now i do not know what to do. I have emailed people from different clinics and finally found volunteer in a clinic here where i am from. but now i am worried about the match. Everything i look up online is for 2018/2019.

i want to make sure I do everything on time and not have to wait another year to do the match process.

I have seen the calenders but it is overwhelming and are not for 2018/2019. when does the process start? What paperwork should i start to look for? I am an IMG, so i imagine they ask for different things.

tantacles

I would recommend you find a mentor who has been through this process as an IMG to guide you through the process. Your school may be able to provide you with a list of people who have successfully matched in the US, and that would be a good start. In addition, SDN forums are a great resource for general information.


failed step 2 cs

I just found out I failed step 2 cs and there not enough time to take it before rank order list deadline. I am applying anesthesia and I am done with interviews. I got a lot of positive feedback from programs and I was told I would be ranked highly by several programs. Rank order deadline is in a couple of days and I have no idea on how I should proceed. I didn't release my CS score on ERAS yet. Please, I need advice. I have worked so hard to be here and I feel like I am about to watch all my hard work crumble before my eyes.

Mr.Smile12

I'm sorry that you weren't able to pass. You need to go directly to your student advisor and seek the advice on retaking your Step 2. The faculty member should have a good idea of what to expect next and whether you have an opportunity to take the exam again before your internship begins.


Can I get into Med School with a major in Communication sciences & disorders, speech/Lang Pathology?

Does major really not matter when applying for medical school? One of my major choices are Communication sciences & disorders, speech/Lang Pathology, but I thin the major isnt going to help me much for getting into med school. Would anthropology be a better major? There's just not as many jobs available for anthropology just in case I don't get into med school.

Mr.Smile12

While a major doesn't really matter, completing the appropriate prerequisites is important. If you're concerned about not getting a job after graduating as a plan B for not getting into medical school, then you can probably do well in the communication sciences/speech-language pathology field. We need a lot of good people who can help patients in that area. You could work for a few years and then if the bug hits you to go to medical school, take more biomedical courses and the MCAT and apply later in your life.


Derm hopeful to Just trying to graduate

I applied to Dermatology with a step score in the 220s and a rank in the middle of my class. I received 3 interviews, a waitlist, and encouraging words. Then just last week I found out I failed step 2 ck. I'm devastated. Reflecting back my anxiety combined with a horrible fall semester (a lot of unusually terrible things happened within that time frame that were outside of my control) had a lot to do with it but I'm mostly afraid because I don't know what to do. I can't study in 2 weeks to participate in the SOAP so am I just jobless with a worthless CV full of derm stuff? Please advise.

Mr.Smile12

I think I state the obvious in saying you need to focus all efforts on passing the Step 2 exam. You need to do whatever you can to graduate and get to the next step. I don't know what your faculty advisor has told you about your chances, but you hopefully were keeping him/her informed about your anxiety and your career desires. Encouraging words help, but you must pass your exams.


Im a DO and want to know my chances in Anesthesia

I took STEP 1 (score is 227) and COMLEX (544). I plan to do 2-4 away rotations and hopefully gain 2 strong LOR's. Im taking STEP 2 and hoping to score in the 240 range. I have very little research experience, but I have a bunch of volunteering experiences. Im also in the military with leadership experience on that end as well. I am wanting to know if I have a strong shot at anesthesia and what kind of residencies should I be more geared towards (top, middle, or low tier)?

-Thanks

tantacles

You have a good chance of matching into anesthesia in general based on your Step 1 score. Keep in mind that away rotations are not usually necessary and may hurt your chances just as they may help them. I would recommend applying to a wide range of programs, particularly since you are a DO, including lower tier, middle tier, and reach programs.


Which is better for IMGs residency in US Vs Australia

What are the best Residency options for an IMG (Indian) US/Australia/Canada and what is the procedure to get in??

tantacles

All IMGs follow the same procedure to gain residency in the United States, and each has its own reputation within the US. I would suggest you ask the schools you are considering for a list of their matches in previous years.

Canada, because of its proximity to the United States, is often a good choice.


LECOM-B vs CUSOM

Hi!

I am trying to avoid putting down two hefty security deposits for the schools and I need everyone's advice to decide on choosing a school.

LECOM-B

PROS: -location - you're not in lecture all day - board scores -older program -affordable

CONS: -no food/water rules - it didn't feel like staff was too supportive . I was told that professors are "discouraged from teaching." Their only role is to facilitate PBL sessions.... which makes me wonder if they will answer any questions students may have about clarifying a concept during a PBL session - Is a PBL curriculum the most effective way of learning all the material in a timely manner? Will I waste a lot of time trying to read through my books to clarify a concept that could have been presented faster in a lecture format? - I'm not sure rotatiosn are currently standing

CUSOM

PROS: - It felt like the staff would put in every effort to see their students succeed. - The SIM labs are absolutely amazing. -video recorded lectures

CONS: - location -cost - variety/quality of rotations? - the program is not as established

I am out of state for both schools. I felt "better" and more supported at CUSOM, but it is a new program. Are SIM Labs really beneficial to your learning experience, or should I be focusing more on rotations? Which school ultimately has better rotations at this point? Any input at this point would be amazing!

tantacles

Every single person who goes to medical school has a different opinion, and everyone only goes to one medical school. If you think that simulation labs will help you and are something you would enjoy, I would suggest you factor that into your decision making process, but your individual priorities may also include cost, education style in the classroom, location, clinical rotation experiences, and many other factors. Ultimately I would choose one school or the other based on these factors as soon as possible to avoid putting down two deposits, but it is difficult to tell you what the right decision is. This will depend on YOUR priorities and not anyone else's.


Are there any medical students/doctors that have had kids?

Hello, I am a freshman (21 years old) and I need to decided whether I want to be a doctor of if I should become a PA. My dad has been putting pressure on me (I think he's unaware of it) to make a decision and become an NP instead because then I can have kids, have a more predictable schedule, and I won't be 35 when I have my first child. I want to have kids around 27-30ish and have calculated I'd be 29 if I choose to do FP, and around 33-34 if I choose a specialty. I know there's a higher risk of having a child with a disability. Are there any students that are pregnant or had kids during residency? Are there any doctors that had kids after you started practicing? Is this something that I should consider when picking a career? Thank you. I know there are different things to consider with each occupation, but this has been bugging me. I am an only child and my dad isn't doing as well as he should be(had heart attack at 50) and I think he may want to see his grandchild before something else happens. This is driving me insane.

tantacles

Many physicians have children. It is very possible to have a child during medical school or residency, and yes, it might mean that you make different choices about your field. However, you can definitely have a child in medicine.


Tips for an IMG from Spain?

Good afternoon,

I am a 4th year Medical student in Madrid, Spain. I have recently been really considering doing my residency in the US. Just to give you a little background, I am one of the top students from my class, doing some research and I am passionate about Cardiothoracic Surgery.

The issue? I know spots for General Surgery residencies in the US are becoming really competitive, hence I have thought of following this path: entering a medical residency program in Spain, and taking the Steps as a resident - tough work, but I'm willing to do it -.

Also, I'd get some USCE in the form of clinical rotations, either during the academic period - many Spanish hospitals have partnerships with US hospitals, and allow for external rotations - or during the summer.

I think this pathway would really allow me to become more confident and competitive in the field I'm hoping to work on, but need some feedback.

-Do you think it's overall a good plan?

-Do competitive programs accept foreign graduates with such profiles? I do know that not many graduates from Spain seek for a residency training in the USA.

-How many months of USCE would you say would get me ready to have a strong application? I know some IMGs get experience by hiring some companies that promise hands-on clinical experience. And I also know some residency programs underestimate the LORs coming from these programs. Is there a better way of getting USCE? (maybe contacting hospitals directly?)

-I have heard about preliminary surgery residency positions. Would it be a good idea to apply to these? Would it likely ensure a spot in General Surgery (categorical).

As you can see, I'm somewhat lost. There is this huge road presented in front of me, and I'm really willing to put in hard work. But I really need some serious advice; info from the internet is sometimes contradictory and hard to trust.

tantacles

Which programs would accept an applicant is largely program-dependent, and many international applicants into surgery do a preliminary year prior to matching into a categorical program. I would suggest contacting residency programs and looking at their websites to get an idea of what they require in terms of USCE as this varies from program to program.


2014 IMG Grad,WANT TO APPLY IN 2020 MATCH. HOW TO DEAL WITH 5 YEARS SINCE GRADUATION FILTER ?

hi, I am a 2015 graduate from India. Have been working under a Cardiologist since then and have been preparing for step1 since few months. plan to give it around Mar-Apr.

My fiance is a 2014 graduate from India. Have been working under an Internist at a tertiary care hospital and has few international publications as 2nd and 3rd author. He has been preparing for step 1 too.plan to give it around feb.-Mar. if we both get > 240 in all steps + first attempts:

1. if we apply for Match 2020, he will be > 5 yr graduate. what are our chances if we apply as a couple vs individually? Both want to apply for IM or one for peds/psych and another for IM , if that increases out chances as couple.

2. how can he get around > 5 yr filter for interviews?

3. what else can he do to increase his chances, if he applies in 2020?

4. how much chances of getting matched even in a low ranking program ,get reduced if he applies as > 5 yrs since graduation(i.e Match 2020) vs just under 5 years since graduation(i.e Match 2019)?

Hope to see a response soon. Thanks.

Sincerely, Dr. Shubhangi

tantacles

These are questions that can only truly be answered by the programs you are applying to. I would recommend that contact some program directors domestically and find out what your prospects are at individual programs as each program has its own requirements.


Medical career lifestyle

Hello my question is which medical career it could be ophthalmology, dermatology, dentistry etc could you rapidly grow your business and expand to other countries. This cosmetic dentist Doctor APA, who has a dental practice office in Dubai and also in manhattan NY travels back and fought which seems like a pretty cool lifelstyle but my question is which medical career could also open such opportunities?

About me : junior in college
Major : mathematics major chemistry minor / premed

tantacles

It is possible to have business in multiple countries as a physician. However, this will depend on the laws of other countries with regard to physician licensure. My suggestion would be to investigate the laws and medical licensure requirements of other countries to figure out your options.


Would H1B Visa status help?

Hi all, I'm an IMG currently on an F1 visa. My employer has agreed to file an H1B on my behalf. Does having an H1B before applying for the residency match have any benefits?

I've heard some programs only sponsor J1. Would it make it easier to get programs to continue sponsoring my H1B since I already have one?

Thanks in advance!

tantacles

Your Visa status will be beneficial at certain programs, and likely will not be beneficial at others. I would suggest contacting individual programs to find out what Visa's they sponsor as each program has different requirements.


Lower STEP 2 CK score (PM&R)

I am applying to PM&R from a top 20 medical school. My Step 1 is 236 and Step 2 CK is 230. I believe my Step 1 score is good (above general average and above PM&R average), but I don't know how to interpret Step 2 CK scores and what weight is placed on them. Would this be a poor, okay/decent, or good score for PM&R? With an above average step 1 for step 1 I thought I had a good chance at top programs, but I don't know if a lower step 2 will decrease those chances substantially? Thoughts?

tantacles

Given your Step 1 score, you should have no problem getting a residency in PM&R. That being said, it is possible that some programs will be displeased with your lower step 2 score, but some programs may not care. This will be very program-specific, and you should not change your strategy based on these scores. You should still apply to a broad range of programs and accept interviews such that your chance of matching based on charting the match statistics is greater than 95%.


Hello iam IMG ,i have a query regarding extension of eligibility period for step -1, i have already extended it once my eligibility period will be over soon but im not in a position to give the exam ,so how can i request for another extension of my eligibility period or how to proceed from here,Thank you

1-if i cannot extend my eligibility period should i start a new application paying the whole amount again?

2-will this be marked in my transcript in future?

tantacles

I would suggest calling the USMLE office themselves to extend your eligibility period; they are the final decision-makers as the organization is private. Only completed exams will show up on your transcript.


Forged ACLS and possible punitive action

I made an egregious error in judgement and forged an ACLS and BLS certificates. They didn't expire nd no patients were harmed. I do not run codes.

I had signed up for the recert classes but when I arrived I was told by EMS staff that they never received my request and would have to reschedule. I panicked and forged the certificates. My job found out and terminated me. Prior to termination I took my BLS. I had my ACLS class scheduled for later that week, but since I had been terminated I took it at another facility. The medical board wants to meet with me after I self reported (I was told my employer would report me to the board anyway). I have been a practitioner for almost nine years and was at my employer for seven. I have never had an issue with my license. I have an attorney but he won't give me a definitive answer. How.likely an I to receive a reprimand or suspension?

tantacles

It is unclear how likely you are to receive a suspension. This will depend on your job and the medical board itself and how lenient they are towards the error you made.


Low step 2 ck score; Should I stop pursuing USMLE

I have a step 1 score of 253 and step 2 ck score of 230 and step 2 cs passed ; all passed in 1st attempt; graduated in 2014 ( IMG); I have one month USCE Should I stop pursuing USMLE cz of my ck... I am applying in this year's internal medicine match but no one is offering me interviews.. Should I forget about USMLE? Do I have a shot at internal medicine residency with this profile.. I have nothing outstanding in CV ..everyone tells me that I am a very attractive person in appearance though if that counts :/

tantacles

Without interviews, you have no chance of matching into internal medicine.

That being said, your Step 1 score is excellent. It might be worthwhile speaking to an adviser so that someone can look at your total application and help you figure out a plan for success. It is hard to say what your chances are without access to all of your application.


Returning to residency?

In 2015 I graduated from an osteopathic medical school and started a psychiatry residency. In the second year of my residency I became too ill to work and had to resign. Since then I have passed the COMLEX 3, received confirmation of PGY1 completion, found a treatment that works and have no work restrictions, and am in the middle of an application for my medical license.

I have too much private debt to make residency feasible at this time. Working with a locum tenens agency I hope to find work in addiction medicine; two years would get me back on track.

Given this background, can anyone give advice on tactics or chances of getting back into residency after two years?

tantacles

It is hard to say how you will fare. That being said, I would suggest that you apply broadly, continue to have clinical involvement, and get great letters of recommendation from your clinical mentors. Getting into residency after taking leave is difficult, and you may have to accept a position in a less than desirable location.


How to Approach Professor for Research Opportunities?

Hello,

I am an undergraduate student in a Dual Admission Program for DO. As a freshman, I've been wondering how to approach a professor for research opportunities? Is it too early or should I ask the professor how I can prepare myself- What do you guys recommend, based on experience?

I have been trying to get close to my biology professor, who is the head of the department at my school. She knows me for sure and we talk but I don't think she knows me well enough to the point where I'd feel comfortable asking her for research. What do you guys recommend I do? Thanks

tantacles

An e-mail is a great way to start asking for help getting research opportunities. Based on this e-mail you can set up a meeting with the professor to possibly do research. When you ask for research, you are asking for a job, so it is not too important how well the professor knows you. I would send an e-mail introducing yourself and expressing your interest in the professor's research.


Step 2 CS

I am an attractive psych applicant but I somehow managed to fail CS. I applied to programs in the midwest. If slots don't open up I will not be able to get a pass before rank lists are submitted. 1) Are there any psych residencies that do not require the pass before ranking? 2) How much will that CS fail hurt my chances of matching?

tantacles

There are many programs that do not require Step 2 CS to be passed before matching. Do your best to retake the exam so that you have a passing score before the match.


Residency chances

Hello,

I'm looking for a honest answer here and hoping someone could help me! I've applied to 155 programs both FM and IM; step 1 224, step 2 244, cs pass 1rst attempt, 4 LORs, all USCE, non-US IMG, need a visa. I haven't received any interviews yet. Do you think I have any chance at residency? and when is it too late to receive applications? I know its really hard to tell because it varies but anything would help Thanks!!

tantacles

You have a chance at residency, but your step 1 score is lower than most IMG's will help. It is hard to stratify your chances, but without interviews, you have no chance to match outside of the SOAP. I would reach out to programs and let them know of your interest.


Study in the US or Ireland

I will soon have dual US/Irish citizenship and am wondering if studying in Ireland would create difficulty in gaining an EM residency back home in the US. I'm originally Irish but have been living here for just over a decade so I'd enjoy bringing my family back for the time I'd be studying.

Curious on what everyone's thoughts on this are as I've not found a satisfactory answer thus far.

Thanks in advance,

E.

Pathdocmd

If you want to do a residency in the USA, especially a competitive one like EM, then attend medical school in the USA. It will be a uphill battle if you are an IMG.


Residency Match dilemma!

Hi, I am a Non US, IMG who graduated 1 year ago and I'd be applying for residency in the US this year. My biggest concern is that I sort of hurried through my USMLE steps and I got a 210 on Step 1 and a 215 on CK while scoring a 260 on my step 3! If I have 1 hands on experience and 1 observership, do I have a chance at matching in Family Medicine or Paediatrics?

Thank you for all the help!

tantacles

There is always a chance. That being said, programs are competitive for IMG's, and it is almost impossible to stratify your chances just based on these numbers; the process is multifactorial and will depend on the program itself and your entire application. I suggest that you post in the forums on SDN; there are many advisers who may be better able to give you an idea of your chances and help you work on your application more.


Considering a Medical Leave

Hello,

I am currently a 2nd year medical student. I have had a history of migraines but they haven't been a problem for quite a number of years until I started medical school last year. I was prescribed nortriptyline by a local PA, which was helping for a while. However, lately I have been experiencing crippling, blinding pain in my head with new and more severe symptoms which unsurprisingly is affecting my ability to function successfully as a medical student. My migraine meds aren't even touching them. I have made it through our first block and we are in the 6th week of school now. I went to the dean to discuss these headaches I've been having and the possibility of a LOA because I'm not functioning very well and have missed quite a few days of school because of them. I thought perhaps I could take a leave to get my health under control and be seen by a doctor (there is NO physician at our school health services - only an NP and no one in the immediate vicinity other than the PA I saw last year.) I was told by the dean that if I were to leave, I would get no refund for the remaining semester even though we are only a few weeks in. I somehow thought that because this was a medical issue - beyond my control - that there might be some way the the school would work with me on this. If I don't get this taken care of - and the school has offered no help or guidance in terms of how I could while staying in school - I am pretty sure I am going to fail out anyway. Since this has started my grades have gone from mid 80's to the 50's, which is unusual for me. I am wondering what the school's obligation is to a student experiencing disabling health problems that are inhibiting performance? Is there a legal obligation at all to help, offer support, or at least reimburse the remaining tuition? On a separate but related note, I have been experiencing some pretty severe depression, for which there is also no support at this school. I tried in vain to get help for this last year (around the same time as the migraines reappeared). There is no psychiatrist here on campus or in the vicinity so I had to return home (4 hours away) to get help. This is not sustainable considering the time demands and workload on a medical student. Considering the climate of medicine and the epidemic proportions of depression/suicide among medical people, this seems blatantly irresponsible. I'm not sure what to do and am frightened of leaving, especially knowing that I will be financially screwing myself, but I am suffering and frightened of what may be wrong with my head. Could you advise? Thank you.

tantacles

The obligations of the school depend on a combination of state law, but the best thing to do is to continue to work with your administration to figure out a solution. It sounds like your school is allowing you to continue later despite not receiving a refund. It may be better for you to withdraw rather than fail out. I would suggest that you take the time off that you need to get your mental and physical health under control before you return.


Will MPH help??

Hi,

 When we applying for residency, If we have done MPH from American University, being an old graduate and attempt on CK. Any help will be highly appreciated.


Thanks

tantacles

It very much depends on the program. Ultimately, all of the other factors that go into residency application, namely step 1 score, grades, and recommendations, will all play a major part. An MPH will probably augment your application but can not save poor performance.


heart failure by Interventional cardiologist?

Can an interventional cardiologist takeup/treat a hear failure case? Would it be any different if seen by a general cardiologist regarding the approach towards the case?

tantacles

Interventional cardiologists may treat heart failure patients as they have done a full cardiology fellowship prior to their interventional fellowship. The approach is provider-specific with some similarities.


Research volunteer at a prestigious cancer foundation vs with a University?

Dear Dr.,

Hi, I am a recent Medical Graduate that is highly interested in Research (Internal Medicine - Oncology). I am planning on spending the coming year working on research to further advance my analytical skills and knowledge before applying for next years match (US). I have found an opportunity with a prestigious Cancer Foundation and was wondering if I should pursue it or instead try to join a research project with a University? What are the pros and cons of each? And most importantly, what would be more beneficial for me on the long run?

Thank you in advance

tantacles

I would suggest that you work on whatever research project interests you. The individual benefits of each project will be dependent on the lab itself. Ideally, you would be working towards a publication at either place, and when that happens would depend on the institution.


Proper Oral Presentation of a Normal Physical Exam

I'm looking for the oral presentation of the physical exam with no luck anywhere. What I mean is the things everyone says when presenting each organ system. For example:

Cardiac = "regular rate and rythym, no murmurs, rubs, gallops....." Pulmonary = "clear to auscultation bilaterally, no wheezes, crackles......"

I'm looking for the complete version of every system. Please help! ????

tantacles

The format varies by institution, but generally has:

Vital signs

Physical exam: General: No apparent distress CV: Regular rate, normal rhythm Resp: Clear to auscultation bilaterally, no rhonchi, rales, or wheezes Abdomen: Non tender, non distended, no hepatomegaly or splenomegaly, Normal active bowel sounds Extremities: Warm and well perfused, no swelling or deformities Skin: No rashes

There are other phrases, but they are not constant in medicine as every place does things differently.


MBA before medical school?

I'm currently a 3rd year nutritional sciences student. GPA is a 3.65 with good EC's. The problem is I'll be graduating college at 20 since I pay for my own school. I'm not sure what the right next step would be. My parents are suggesting that I get an MBA and then apply to medical school so I'll be more of age. I've always pictured myself as a doctor but I'm not 100% sure if medical school right for me. Would an MBA be a good next step whether or not I want to go to med school after? Thank you

tantacles

If your application is good and you are able to score well on the MCAT AND you have all of the necessary extracurriculars, it is completely appropriate to apply right when you finish college or a year after without getting another degree. It's also acceptable to work for a few years if you need money to pay for your application cycle. No one way is right, but be sure you only apply when your application is truly ready for medical school.

An MBA will teach you little about medical school and will ultimately put you more in debt. My suggest is that if you are unsure about medicine you find a job in the medical field; this will give you more experience in medicine and allow you to make a more informed decision, killing two birds with one stone, as it were.


Fellowship

I am about to finish my specialization as a pediatric surgeon in Colombia. I graduated from the national university of Colombia and i would like to do a fellowship in fetal surgery in te US. I have dual citizenship with the USA and Colombia.

Would I be able to do a fellowship in the US and if so what would i have to do?

tantacles

If you wish to do a fellowship in the United States, you must do a residency in the United States. While some programs will allow you to backwards train, doing your fellowship first, most programs will require you to do a general surgery residency followed by a pediatric surgery fellowship, meaning that you would have to go through the residency application process, taking USMLE step 1, 2, and 3 and apply to residency through ERAS. I would suggest you find connections in the United States to help you figure out your options. Pediatric surgery is a highly specialized field with few specialists in the United States, so you will need inside advice to figure out your realistic chances.


refrences to study

i am just about to start going to hospital as an extern.and i will start it with internal medicine.the kind of refrences medical students mostly use in here looks like not to be useful enough to me .so i am here to see what other peaple around the world study and to ask you for help .it would be perfect if you could help me introducing the best or to help me choose because i have not read the books and i dont have any idea about them. for emegency medicine is tintinalis a good choice? usmle step2 ck and harrisons handbook for common diseases.any advice? torento notes? i have just heared about these books . i appreciate any help. thanks in advance.

tantacles

On my internal medicine rotation, I mainly focused on answering prep questions as my university required us to take an exam after our rotation. I used USMLE world.


Does Harvard School of Medicine accept people who applied to other med schools at the same time?

I want to attend Harvard Medical School as a first-year student but their website says "PLEASE NOTE: Students who have been enrolled in medical school are ineligible to apply for admission to the first-year class. Students who have applied for admission to HMS on two prior occasions also are ineligible. HMS does not accept transfer students." Does this mean I can not apply to other medical schools at the same time I apply for HMS? I wanted to apply to more than one school so I have a better chance of getting into medical school that year.

Mr.Smile12

You can apply for admission to multiple medical schools through AMCAS. The warning is to let people know that you cannot transfer once you have been admitted, matriculated, and enrolled for courses at another medical school and want to go someplace else. If you have specific questions, ask them.


Would you trust your classmates as your doctor?

So for those that are past their first year in medical school, I assume that you have a pretty good idea of what your classmates are like. My question though is that based on your own personal experience, would you want them to be your doctor? Would you want them to deliver your baby? Would you want them to be the one that does neurosurgery on you?

    • This is irrespective of their performance in medical school, because everyone knows preclinicals and clinicals does not translate to one's ability to be a doctor. I'm just asking from your own personal experience with them.

In my opinion, there are a lot of people in my class that I would trust and be ok with, but there are certainly others I would not want touching me or my family.

tantacles

There is tremendous variance in the quality of physicians coming from all schools and all backgrounds. There are some that I would be delighted to have as a physician, and others I would find difficult to interact with on a professional level.


As an international medical graduate, what sort of extra curricular activities should I participate in in order to get a Gen. Surgery residency?

I went to medical school in India and am currently studying to write the USMLE Step 1. I was wondering what sort of extracurricular activities I should participate in in order to help me getting a Gen. Surgery residency.

tantacles

I would suggest that you participate in research related to surgery and do as well on your board exams as possible. You will also need to do preceptorships where you get US medical experience as an observer.


Best way to use your time in hospital

I am a fourth year medical student and 1st of september is going to be the first day i am going to hospital as a stager. I find lots of my friends who are going to start their internship regreting about their past 2 years being wasted.one thing they all say is about the fact that the priority of doctors in our hospital is not to teach us but its to cure patients .i had heard it alot that students must learn everything bythemselves and thats what really concerns me.any advice ,experience ,apps(i had recently installed medscape and found it so helpful)is so helpful.thanks Sorry for my bad english..

tantacles

I would suggest that particularly if your hospital has access, you use sites like uptodate to look up any patients that you see. This is most likely the best way to learn, particularly if attendings are not able to teach you.


Please let me know your thoughts on this dismissal

I go to a DO school. Last year, I started school w/ class of 2019. I started feeling sick at the end of block 1, and had huge problems w/ getting up to go to class and would often ask for extensions on assignments, bc of inability to complete them - turns out, I had mono. I went on medical leave at middle of block 2, but wasn't diagnosed until April of 2016, since the mono was caused by CMV and all my docs were looking at epstein-barr, lupus, rheumatoid arthritis, etc. I requested testing for CMV (PCR & serum) and both came back positive. When I was leaving school, I was obviously an emotional wreck, since I didn't know what was wrong with me but just felt overall horrible.

Flash forward to now: I was allowed to start back to school w/ the class of 2020, but only after signing a contract saying that I wouldn't be tardy to anything, miss anything, and that required me to be in counseling throughout the year (since the administrators saw me upset, crying, and generally being a sloth in the process of going on medical leave). I was also put on academic probation from 2015, and wasn't allowed to get off of it despite starting out a new year. Basically, I had to sign a "learning agreement" because my behavior while I was sick wasn't acceptable healthy medical school student behavior. I signed it, because I wanted to be back in school and didn't challenge any of the points in the contract.

I did really well in Block 1 of this year, but had to remediate a class in Block 2 & 3, which per the handbook, is totally allowed and I passed each class and was ready to tackle block 4.

Block 4 started out rough, I had gotten a vicious stomach bug, stayed in the ER overnight for suspected appendicitis, and was diagnosed with a paralytic ileus. It put me out, and it was no fun, obviously. Basically I missed a week of school because of it and it put me REALLY far behind, farther behind than what I realized until the second test of cardiopulm physiology.

Ultimately, when I realized I was behind and having trouble catching up, I emailed our academic success department about scheduling a meeting so that I could nail down a study plan and get caught up. I also emailed our medical education department several times about discussing grades and additional more boring subjects, to which I received no response. I actually had to go into the med ed office to see if they were even receiving my emails, to which they said yes, but they receive so many, that basically the one I sent got sent down to the bottom of their list to reply to. :O I reached out for help from classmates, brought my grades up from the mid sixties to upper nineties, adjusted my study habits, etc. etc. but in the end, it wasn't enough to bring 3 of my grades up from the mid sixties, and 2 from the high sixties. Regarding how I brought my grades up, in my appeal, I had two fellow students write me letters of support, one from my tutor and one from the class president.

The school wants to dismiss me, based on what they said were academic reasons (understandable), but they also cited the absences from LAST year from when I was sick w/ mono as a reason for dismissal, along with my MCAT score and undergrad science GPA. In the letter that outlines why I am being dismissed, it clearly states that "you are being dismissed purely for academic reasons." I am unsure of how my MCAT score, absences from being sick, are even worth adding in to the letter if it's purely because of my academic performance while in medical school. Also, the school let me in with my MCAT score and undergrad GPA, and now they're holding it against me? I get they are trying to build a case that I am academically incompetent, but they are the ones that let me in.

In the dismissal letter, they also stated that I failed a course that I didn't fail (I brought this to their attention, and they rescinded it and said they were misinformed). This makes me raise an eyebrow - they had written my dismissal letter based on faulty information - how the heck were they misinformed about me failing a class? The class they said I failed, I actually did really, really well in.

They also didn't bother addressing the information that I provided about how I continuously reached out to the administration put in place to help students and how they flat out just didn't respond. I seriously believe the school is holding a grudge from me getting sick last year and how I was a pretty crappy student when I was sick - I know it sounds crazy, but the fact that I had to broadcast on social media to my peers that I needed a tutor this past block (bc the school wasn't helping), seems a bit absurd, too. I have gotten the feeling that administration is annoyed by me from the moment I got back from medical leave.

I know that I am capable of succeeding as a student and that getting mono last year was inconvenient for everyone, but I also know that starting out this past block getting behind and scoring poorly - but taking responsibility for it and bringing my grades up from, in one class, a 43 to a 95 (!), hopefully demonstrates that I am driven and will do whatever it takes to make sure that I am able to learn the material. I laid out a study and remediation plan in my appeal letter and let them know what I did differently to bring my grade up from absolutely atrocious to the mid-nineties.

Ultimately, I take issue with a) being dismissed in part because of absences that I earned (which, by the way, were in the allotted amount allowed to be missed) when I wasn't able to really even get out of bed - and have documentation from four doctors that I had mono - these were all given to the school b) dismissed, in part, when I was obviously an "at risk" student, but even in light of asking for help from appropriate people, I didn't even receive a response (in the student handbook, it states that at-risk students, who DON'T progress academically, could get dismissed; however, I wasn't given a procedure through which I could academically progress and had to figure it out on my own). I get that my grades at the beginning of this past block were horrible, and if that were the sole reason, it would be a bit more understandable, but since there seems to be more to it than that, I just want insight.

I know that I have shown the school that I am dedicated to the profession AND am capable of bringing my grades up, even when getting behind (I also revamped my study techniques and have now obviously determined what I need to look out for in regards to getting behind in the first place) - and even when the resources in place don't respond to emails inquiring about meeting for help. What would you guys do? I have appealed to the dean, but his response genuinely leads me to believe that he, at most, skimmed my appeal.

Has anyone ever won an appeal based on similar information? Is this a "fair" dismissal, based on the information I have provided?

tantacles

It is hard to say whether your dismissal is fair as we are only hearing one side of the story - yours. I don't think that on this site we will be able to give a satisfactory answer with regard to that question.

I do think, though, that if you want to pursue medicine at this particular school, the services of an attorney might be helpful.


Best way to use your time in hospital

I am a fourth year medical student and 1st of september is going to be the first day i am going to hospital as a stager. I find lots of my friends who are going to start their internship regreting about their past 2 years being wasted.one thing they all say is about the fact that the priority of doctors in our hospital is not to teach us but its to cure patients .i had heard it alot that students must learn everything bythemselves and thats what really concerns me.any advice ,experience ,apps(i had recently installed medscape and found it so helpful)is so helpful.thanks Sorry for my bad english..

tantacles

I would suggest that particularly if your hospital has access, you use sites like uptodate to look up any patients that you see. This is most likely the best way to learn, particularly if attendings are not able to teach you.


Graduated from medical school 2 years ago

I appreciate any insights and advice you may offer. I graduated from medical school 2 years ago and am no longer in contact with any of the admin staff there. Following medical school I joined a company and have been employed there. I decided not to pursue residency because I get burnt out and depressed during my clinical work and felt little passion towards continuing in that path. Since then my passion for medicine has reemerged and I'd like to consider my options in applying to residency programs. I know it may seem like a non shot or not possible at all but I'd like to revisit this before my step scores expire for good. I was an outstanding student with high step and shelf scores, for what it's worth. If there is a chance at applying and getting interviews, what are the next steps?

Thank you again

tantacles

You will need to contact your school and express to them that you wish to apply. They will be able to help you through this process.


How was your experience during general surgery internship, general surgery residency, and/or trauma/critical care fellowship?

I'm only a high school sophomore so correct me if I am wrong please, but I believe that the three I have listed are the things you do right after graduating from medical school if you're planning to become a trauma surgeon. Even if you're not specifically going for becoming a trauma surgeon, these are still part of the path for other medical specialties I think so I just listed it separately.

For the sake of helping people, I've been wanting to work in the ER. I found out that the type of things I want to do are what a trauma surgeon would do, not an EM. Here's the thing, though...

I'm terrified.

I'm not afraid of blood and all that. What is terrifying is things like a surgeon shouting at you during your internship while an emergency surgery is taking place, meaning someone's life is currently at risk. Or, completely having no idea what to do when you're the leading surgeon and everyone in your team is depending on you for correct judgement and orders.

I could freaking choose other medical specialties that would be less terrifying than surgery. Although any medical profession isn't easy, I'm pretty sure there are other types of paths with less pressure compared to the pressure during an emergency situation. Those other paths could even have a higher salary. And yet, I really want to be a trauma surgeon. I feel like my fears are shouting at me.

So, I think I could get a better look on this if I heard people's experiences...

tantacles

Many of these decisions would be best made during medical school. During medical school, you will have a chance to rotate through multiple surgical fields and see if the field of surgery is right for you. At this point, I think you should focus on getting into college and doing some volunteering in a hospital. When you are of the appropriate age, consider getting training as a nursing assistant or EMT so that you can get some experience working in medicine, which will both give you better insight into medicine and improve your application should you decide to apply.


When i have to study step 1

I am a medical student in 3rd year ... I just wanna to ask about when i have to study for step 1 usmle ... Is it necessary to take it ???? How much it is difficult??? And whats the advantages for it???

tantacles

If you wish to match in a US allopathic residency, you must take USMLE Step 1. Otherwise, it is not necessary for you to take the exam. The difficulty of the exam depends on your skill in taking standardized exams and your level of preparation.


What schools should I apply to? Are my chances good?

I have taken the MCAT 6 times (25, 23, 498, 498, 502, 511 in that order) and have a 3.70 GPA from Johns Hopkins University. l am also an URM (Puerto Rican) and a reapplicant (my new mcat score was very recent) I was wondering what schools you think I should apply to and if my chances are good for this cycle. I am in the process of getting verified by AMCAS.

P.S. I have done over 3 years of research (the most recent in the best HIV lab in the country at JHU) and have my name on 3 CROI abstracts and am in the process of getting published. I also have a lot of clinical shadowing and clinical experience, as well as a ton of community service.

tantacles

Your chances based on MCAT and GPA alone are reasonable, though your history of sub-par scores may hinder you a bit. I would suggest you apply broadly to schools with a wide variety of competitiveness. Use the Wedgedawg application tool, available for free on SDN, or purchase the MSAR for $28 and use it to select the appropriate schools that fit your MCAT and GPA.


Paranoid needlestick?

Basically. Wanted to know if anyone else had this experience. Was poked/scratched by residents pickups in OR during durgery. There was no hole in outer or inner glove that I could see, but still had scratch on finger. No blood. Paranoid about needle stick.

tantacles

If you are worried that there was exposure, I suggest that you visit occupational health at the institution you study at to find out if you require some prophylaxis.


Will a PharmD hurt my chances of getting into medical school?

I am currently an undergraduate on a pharmacy track. However, I have realized that I am truly passionate about medicine and want to apply to medical school. Should I finish my pharmacy degree or should I switch into the pre-med route? More importantly, will med schools view a PharmD unfavorably? As long as I can maintain my good GPA and do well on the MCAT and explain myself well, will my chances of acceptance into medical school be just as valid as someone with a BS?

tantacles

A pharmD will not hurt your chances of getting into medical school.


Malpractice insurance

Is there a way to arrange a malpractice insurance for a foreign medical student doing clinical rotation in Columbia University? The school told the student he would have to arrange it himself.

Thanks

tantacles

This is usually arranged by the school itself or the student is otherwise not allowed to participate in the care of patients and comes as an observer.


Demographics

Has anyone made a list of demographics or found a resource where Age, Background...etc. are all combined together in a table or maybe online. I keep finding myself making mistakes on some demographics that were not listed in the First aid or pathoma and there was no other real clues in the question stem to help me rule out the other answers. Please i am taking my test in 3 weeks and i need your help and guidance. Also i need an advice on the best sources for pathophysiology and biochemistry. I realized i need more detailed readings. I am at a point where i feel like i know a lot but not whats leading me from point A to point B. And there's a handful of questions that i missing because of that, whether it was pathophysiology or biochemistry. Thank you. (i use FA, Pathoma, Uworld)

tantacles

Many students have posted their study strategies online. Myself, I used pathoma, First Aid, and Uworld exclusively, and I found them to be perfectly adequate for the purpose of the exam. I would also suggest that you take plenty of practice tests.


Residency Program Views on Extension of Step 1 on MSPE

I decided to extend my Step 1 exam after attempting (and failing) to simultaneously juggle my own wedding and studying. I recently found out that in extending my Step 1 exam, this will show up on my MSPE when I apply for residency programs. I will only be extending my Step 1 exam four weeks, so my graduation date will still be the same. I'm curious how negatively residency programs, particularly surgical residency programs will view this extension in my Step 1 exam showing up on my MSPE even though my graduation date will not be effected. Thank you!

tantacles

Most programs should not care too much as long as you did not fail on your first try. Just do your best to get a great score.


Residency matching for PhD dropout

How would residency programs feel about senior medical students who has started in a PhD program prior to medical school matriculation but never finishes it? Would it work against the applicant? If yes, would it make any difference if the applicant finishes with a master's degree and have a closure, instead of being a "PhD dropout"? Would it depend on specialty?

tantacles

It depends on the student. If you did not finish the degree, you do not need to mention the lapse in your CV as you need not reveal your age. You can simply write that you have a Master's degree and not write about your PhD if you wish.

I doubt the dropped PhD will have much impact.


Observership in canada

Hi I'm the 3rd year medical student from India, I want to do the observership in canada (Edmonton) . I wanna know the time period of course and will I be able to do it just being the student or els I have to wait till the completion of my MBBS.

tantacles

My suggestion would be to contact the individual hospital your are interested in doing an observership at. Most hospitals have their own policies, so determining an exact place to rotate could be helpful.


Mentioning my Anxiety Disorder in my Psychiaty PS?

Hello,

I am an MD and will apply for psychiatry residency this year and I have a question regarding my PS. I was diagnosed with Generalized Anxiety Disorder at the first year of my Medical School. With this experience, I understood how patients feel like, what they are going through, how psychological pain is true and sometimes more painful than physical pain. I gained a great understanding through mental illnesses. It made me more emphatic person. It allowed me to make sure I want to pursue a career in Psychiatry. Now I am not sure if I should write this on my PS. Does it show me a weak person and can't do well in stressful situations? Or does it make a positive impact on PD's?

Thanks!

tantacles

If you are able to portray your mental illness in a positive way, it is completely acceptable to mention your mental illness in your personal statement and mention it as a motivation. How it will impact your application will depend on the quality of your writing and the way you portray your illness. I would aim to show the positive impact your illness has had on your medical education in some way.


How difficult is it for a DO to match into an I6 cardiothoracic surgery residency?

I am currently accepted to a DO school. I never applied MD because I was accepted early to a DO school. I graduated undergrad with a 4.0 and have a lot of clinical experience. Some people are telling me I should drop my acceptance and apply MD for the next round. How much of a disadvantage am I at trying to match into one of the I6 residency programs being a DO? If I manage to get good grades and do well on the steps does it even matter where I went to school?

tantacles

You will be at a great disadvantage for these positions if you go DO. However, if you drop your DO acceptance, you may not be able to apply to DO schools successfully again. I do not know your other statistics so it is impossible for me to advise whether you'd likely be accepted to an MD school.


Chances for a DO medical school

So I have a problem; I'm a worrier, and I was wondering if anyone can give me some tips on how to improve my application for DO medical schools. I guess I'm just wondering if I can make up for my low GPA. :(

-My current cumulative GPA is approximately around a 3.6, but my science GPA is at a 3.3, 3.45 if I do well on my finals. I've already taken the MCAT, but I don't know what my score is yet.

-I've held two officer positions, one for one semester, the other for approximately 1.5 years, both of which I've learned a lot from.

-I've done research on HIV and psychosocial issues, and my professor is including it in his documentary (does that count as being published????), and am doing it for my research dissertation.

-I've published poems in magazines and journals.

-I've volunteered approximately 300 hours through making blankets for the ER hospital at Medical City for 9 months, a volunteer mentor at a local elementary school for two years, and a volunteer tutor at my university (was named volunteer tutor of the year) for six months, and volunteering at two nursing homes, one for two months, the other for five.

-I've worked as a medical assistant for six months, and have shadowed four doctors, which brings my clinical experience up to around 400 hours.

-I'm working on my personal statement, and with the help of my professors and friends who got into medical school, I think it'll be okay. But what I'm really asking is if I have a chance of getting into a US DO school??????

-Again, I love you all, and I hope to contribute to this forum as much as I can! I apologize that the first question I have for you is a little desperate, but I'm applying this cycle, so...

tantacles

If you do very well on the MCAT, you not only have a chance at DO schools but also a good chance of garnering acceptance to MD schools. I would suggest that you purchase the MSAR and take a look at the Wedgedawg application tool on SDN when your MCAT score comes back (I can not tell you what your chances are without an MCAT score!), and solicit advice on the What Are My Chances forum on SDN if you are still unsure. You have overally done well in your undergraduate, so do not sell yourself short if the MCAT went well!


Faild step 2 cs

I'm an IMG i took my step 2 cs , it was my first step Few days ago the result was resleased and i found that i have faild in the CIS part ( even i have done very well according to FA for cs ) but i dont know what happened exactly i'm totally shocked And i'm now wondering how would that affect my chances for the match in the future please help me .

I have only one month hands on USCE , if i do more USCEs or research will that help ?

tantacles

Failing Step 2 CS will most likely affect your chances of matching to residency. It is difficult to say how much it will affect your chances, but it is important that you pass the exam on your next try. It is important that you offset this failure with successes in research and more clinical research.


step 1 score release

i took my step 1 exam on april 6,2017, should the expected scores be released tomorrow? usually what time? thanks

tantacles

It is uncertain when exactly scores will be released tomorrow, but the scores are usually released in the morning.


General surgeon and a neurosurgeon?

Is it possible to become both a general surgeon and a neurosurgeon?

tantacles

It is absolutely possible to do both. Keep in mind, though, that this process would likely take over 10 years as you would have to do two residency programs separately.


Help choosing a school: Albert Einstein versus Northwestern (Feinberg)

I am having trouble deciding between two schools, and the deadline for choosing is fast approaching! The dilemma is that I want to go to Einstein because it is close to my family and where I grew up (and also because I just like it), but some people have advised me that it is a bad decision to turn down the opportunity to go to a top 20 school (Feinberg).

So my underlying question is, how much does ranking/prestige matter for my future medical career? By going to a lower ranked school, am I giving up on having a better chance of matching into top residency programs in the future?

Another component of this question is, if I know I want to do residency in NY, is it actually smarter to go to Einstein because it is in the area? Some say it is easier to match in a city if you went to medical school in that city, but others say going to a top 20 school would have more of an impact.

Some possibly relevant background info is that I know I won't want to go into a super competitive specialty like derm or ortho...I'm pretty set on pursuing psychiatry, and if not psychiatry then a primary care profession. Also, I do like research but am definitely pursuing medicine in order to practice and not to go into academia. These are some of the reasons I feel that going to the most prestigious school possible isn't as important for me as it would be for some others (?)

Any advice on what I should be factoring into this decision-making process would be greatly appreciated!

tantacles

Ranking and prestige only matter insofar as being at a highly ranked residency program matters to you. For some people, the ranking of their residency program makes a big difference in their lives. For some people, they just want to be a doctor and be the best they can. You've gotten into two schools that are excellent and that will both be able to propel you towards careers in academic or private practice medicine, and if you do well in medical school, the sky's the limit. So my suggestion is that you choose the school that you like the most and that is least expensive. The schools you are accepted to are fantastic and will not limit you at all.


Been feeling inadequate...

I have just embarked on my dedicated prep period for my Step 1 and my stress and depression has been through the roof. I don't suffer from depression but I have been feeling down, angry and really pessimistic. I took my school administered CBSE exam 3 times before I just barely passed last week. It's good news and all, but the score equated to a just barely passing Step 1 score. I'm really feeling inadequate and that I won't be able to achieve the score I need to achieve. I ask myself more times than not, is this really meant for me? I just cant seem to keep up with peoples stellar scores.

To top it all off, I can't seem to talk to my fiance about any of this since shes a medical student too who doesn't seem to understand my plight. Also, she seems to get affected by my negative-self talk/venting so I stopped talking to her about those things. Now, I feel all alone and I reached out to a therapist as of late and overcome with fear over a future PD erroneously throwing my application into the trash bc I had been treated for mental health related issues. I'm not really SIGE CAPS positive, but I'm really not feeling happy at all. HELP :(

tantacles

It is unlikely that anyone on SDN will be able to fix your problems. I think it is great that you have reached out to a therapist. Your emotional well-being is important. As far as Step 1 goes, the most important thing is that you pass on your first try, so keep studying up until the exam and just do your best. Take some practice tests on your own to gauge your progress, and look into some study strategies posted on SDN to help you out.


how many mistakes allowed on step 1 to achieve 260+?

I took my usmle step 1 on april 6, 2017, and i was wondering how many mistakes (ive counted 11 so far) can someone make and still achieve a 260+? thanks

tantacles

The number of questions you can get wrong on Step 1 and still get a 260 is dependent on the individual exam administration and not released by the body that provides the exam.


How long does my board review usually take? (NAVY HPSP)

I have already completed MEPS in November and I have talked to my recruiter approximately 6 times since then. He keeps saying that I am still pending board review. It is now April and I start medical school here in July. Does board review usually take this long? Do I have any reasons to be worried?

casedentalmed

There may be a lot of reasons for the delay that have nothing to do with your qualifications. Hopefully your recruiter is keeping an optimistic perspective for you, but it is hard with the state of federal funding being what it is. There are a lot of administrative decisions and factors made that ultimately determine how much money and scholarship spots are available.


Love EM, but absolutely HATE night shifts + random schedule. Need Advice!

I am drawn to everything about the actual work of EM and found that I fit in with people in the field more than any other. The only problem I have, is I hate overnights and changes in schedule. I am one that likes a regular routine schedule- not necessarily while at work (i like the randomness of the ER), but as far as when to go to work, sleep, and such. I also have concerns about family etc with some of the horror EM schedules I hear of. With that being said, as an undecided M3, everyone keeps telling me to "do what interests you the most" and "do the field you would be happiest in". When i sit back and think about the type of physician and work I would like to be doing, EM fits this perfectly in many ways- acuity, procedures, knowing a little about everything, helping to save lives, culture and people in ED... i even like the psych and drug overdose patients that so many in the field of EM complain about. I just dont know if i can live with such an erratic schedule for the rest of my career :( I am not as concerned about pay as many others, so i would be happy to take a pay cut if i obtained more normal hours, but is this even possible right out of residency? I know I will have to go through this tough schedule during residency, which is fine and I believe i would handle it ok for 3 exciting years. My main question is if after residency, it is possible to have more "normal work hours" if i am willing to take a pay cut? I fear that the scheduling in the field will make me dislike it and regret not going into something different, but at the same time i dont think id be happy in another field. This concern is huge, so if it has that big of an impact on me, should I disregard EM?

tantacles

It is, of course, possible to have a normal schedule after residency. Keep in mind, though, that your overall happiness during residency matters, and it is important that you take your preferences, whether or not they are related to the specialty, into consideration.


What are my options if I don't match in Radiology ?

Hello,

I'm an international medical graduate who is currently doing research in radiology in the US.

I would like to match in radiology. This year while I applied 2 months late and got only 1 interview I did not match. However this increased my stress as to the possibility of not matching next year and possibly ever.

As a result I am thinking about trying for my next 2 favorite choices: FM and IM. However I am afraid that my CV and experience being now too focused on Radiology I also won't match in these 2 residencies

Is it still possible to match in FM and IM given my situation ? if not what can I do to change that ?

Thanks a lot!

Marwen

tantacles

If you wish to match in a different specialty, you would likely need to have more activities related to internal medicine and family medicine. However, note that you can send letters of recommendation separately to each specialty, which would increase your chance of getting interviews. You simply submit only internal medicine and family medicine letters to those respective specialties and omit your radiology letters.


Should I become a MD or Do anesthesiologist? what's the difference? ?

Lee

There are two pathways to become a physician: MD or DO. Both have equal practice rights throughout the US.

Generally the question is first, what school can you get into?

Once you're in an MD or DO school, then during your 3rd year you can start looking at applying for anesthesiology residency programs.

Usually it's easier to get into a competitive residency with an MD degree. However, there are many anesthesiology programs to choose from -- MD or DO you will get into one as long as your medical school grades are good and board scores are decent.

Pay and practice is the same, MD or DO.


Judging from the difficulty of reaching the COMLEX scores required to successfully match into dermatology residencies, should I still aspire to specialize and do research in dermatology as a to-be osteopathic medical student?

Currently enrolled in an accelerated BS/DO program, and likely to stay with it. Is it near improbable for a D.O. to match derm?

tantacles

If you are interested in dermatology, I would strongly recommend you do research in it. Any research you do will be valuable and will assist you in getting into residency.


saint james school of medicine - caribbean

This thread probably been discussed a lot regarding Caribbean med school vs. Us. Different caveat...Does anyone have any opinion regarding saint James medical school (https://www.sjsm.org/)? Its very economical comparing to any medical college in US even comparing other Caribbean medical school like Ross. but what's the catch? Would it be completely useless MD degree without having any chance of getting into residencies?

To their defense saint james says, its up to the students how they want to use the information. If someone goes through the MD program and does very well in USMLE step1 there will be no issues in securing residency. What does the critique think? A residency director looking at someone with a decent Step1 score but from saint james. Deal breaker?

tantacles

The fact that Saint James is more economical doesn't change the fact that it is still a school in the caribbean. I strongly suggest you read the following thread on SDN, which explains some of the pitfalls of going to a caribbean medical school.

https://forums.studentdoctor.net/threads/caribbean-md-or-crna.1191947/#post-17600322


Is it possible to get into an IM residency and then switch to dermatology residency?

Since I am an international medical student aspiring to be a dermatologist in the U.S. my question is, is it better to do an IM residency and the switch to a dermatology residency ? Or directly apply for dermatology residency ( I have been told the chances are very low to get one directly ) Would IM Residency look good or bad when applying for the dermatology residency?

tantacles

It is always better to directly apply to a residency program that you are interested in; Internal medicine residency is 3 years long, and while it may make it slightly easier to be accepted into a dermatology program, programs looking favorably on this scenario will likely not advertise this. Your best shot would be to do electives at the institution at which you do internal medicine residency and be an absolutely stellar resident, which might result in the program knowing you well. However, I would not advertise this to any internal medicine programs you attempt to match at as they likely want residents who are interested in their field.


Is it easier to apply for an IM residency and then switch to dermatology for IMGs who ultimately want to be dermatologists? Since getting a dermatology residency directly is harder?

For interaction medical students who want to be dermatologists in California, what steps should be taken? Also if one has an IM background trying to switch into dermatology, is there any advantages of having an IM background ? Does it increase their chances of getting into dermatology or decrease their chances ? Since I'm an international student who wants to work as a dermatologist in the U.S., have receive only bad news about how impossible it is.

tantacles

It is always better to directly apply to a residency program that you are interested in; Internal medicine residency is 3 years long, and while it may make it slightly easier to be accepted into a dermatology program, programs looking favorably on this scenario will likely not advertise this. Your best shot would be to do electives at the institution at which you do internal medicine residency and be an absolutely stellar resident, which might result in the program knowing you well. However, I would not advertise this to any internal medicine programs you attempt to match at as they likely want residents who are interested in their field.

Indeed, as an international student, matching into dermatology will be very difficult. It is such an intensely competitive field that even US allopathic graduates have trouble matching into it.


Mistake in Personal Statement

I am so worried. I made such a big mistake. I have written the name of NYU College of Global Public Health as NYU College of Global Health!! Three times!! I am so dead! Even though I have GPA 4 and good recommendation, this still is a huge mistake! What can I do?

tantacles

Unfortunately, since you have submitted your application, there is nothing you can do. The mistake isn't huge, and an admissions member might not even notice. I would suggest waiting and seeing what happens. If you aren't accepted to this one school and you are intent on getting an MPH, presumably you can go to another school that accepts you.


USMLE: Are my books too old?

I'm an IMG and plan on studying for step 1 of USMLE. I have a First Aid 2011 and Kaplan notes from 2008. Are those book too old to be using? Should i invest in newer editions?

tantacles

The Step 1 exam has not changed substantially in the last 6 years in terms of content, but it is possible that there are more errors in the book that you have now than in a more current edition. My suggest is that you purchase the 2015 version of these books, which you can likely get at a discount on an online store.


MBA in pathology

I'm a pathologist who is AP/CP certified and was wondering whether doing an MBA will help advancing my career in the pathology or not. Any thoughts ?

tantacles

It is hard to say. If you hope to go into a career in business, particularly insofar as it's related to medicine, that would be helpful. However, I would suggest that you think about the kind of job you are looking for, look at leadership in that position at multiple hospitals, and see what degrees these particular people have. That should inform you of whether this degree will help you.


What does generic email of PD to consider them highly, before match means?

I got an email from new PD of the program where I interviewed, and mentioned me to consider their program for the residency and they ensure to provide the best training ( seems generic though). What does it mean? Have they ranked me high enough to match ? How should we respond in such case ? Please suggest me.

tantacles

You do not need to respond to these emails at all. My suggestion is that you rank the programs in the order that you like them the best. A program director's email should NOT influence your match list. Match communication has historically been a poor indicator of whether someone will match at a particular program.


How often can one change the date for USMLE step 1?

I am taking the USLME step 1 this year and I changed the date once already; how often can one change the date for the USMLE step 1. I am just not ready to take it in 6 weeks.

tantacles

You may change the dates as many times as you like. Just keep in mind that the testing center will charge a fee if you change the test less than 30 days before the testing date.


Does getting accepted into a MD program in US will guarantee a residency?

Anyone knows if there are people who got accepted to a MD program followed by reasonable scores in step1 and step2 but were unable to secure a spot in residency? Even primary, internal med or not that competitive residency slot? How many people are out there with a nothing to do MD degree without getting a residency? Not talking about a match. Even a mismatch,...does most if not all get accepted in some sought of residency once they get in to MD or DO school in US and get average step1 and step2 score? How many unemployed doctors out there?

tantacles

There are, in fact, people who are unable to secure a spot in residency after getting accepted to an MD program. The reasons for this are varied, but most of the time come down to poor board scores, failure of classes, and/or choosing a specialty that one is not competitive for given one's profile. The vast majority of medical students get into some field of medicine after attending a USMD program, and these few are outliers. Most US MD schools boast match rates above 90%.


undergraduate engineering degree for admission

If undergraduate gpa is lower than average but undergraduate major is engineering, does the admission committee scale the gpa differently or consider as a stronger candidate in comparison to someone with a higher gpa but in Art degree or regular science?

tantacles

Generally, admissions committees may consider the rigor of an undergraduate major in selecting candidates, but a 3.9 will always garner more praise and attention than a 3.5. Medical schools see applicants from hundreds of different undergraduate institutions, and it is impossible to figure out the GPA of a particular major or even the average GPA at a given school. I would suggest that if your goal is to go to medical school that you choose to major in something that you're interested in that will give you the highest GPA possible.


DO school or Caribbean for opportunities in MD residency

If someone has to pick between DO school in US or Top Caribbean school, which option has better chance to get into MD residency program given equal Step1 and Step2 score?

tantacles

A DO school is the superior choice at the beginning of medical school. Much has been written about this topic, but suffice it to say that Carribbean medical schools inflate their match rates. They also count prelim matches as successful matches, and those positions may or may not result in further training depending on the trainee.

DO schools, on the other hand, try to graduate all of their students and have high match rates each year.


GPA of undergraduate or GPA of graduate work. Which is more important for application?

I have cum under graduate GPA of 3.1 but graduate GPA is about 3.4, much higher with increasing complex courses. How the admission committee will look at this? Focus more on under graduate GPA or see the steadiness and complexity of graduate experience and take that into the factor more?

tantacles

Your undergraduate GPA is by far more important; it is the only piece of your GPA that goes into your AMCAS GPA and is the most easily accessible piece of information, along with your MCAT, to medical schools. If you have a GPA of 3.1, I would suggest a post-baccalaureate program to improve your undergraduate GPA and show that you are able to handle the rigor of medical school as you are unlikely to garner an acceptance without an absolutely stellar MCAT score, and even then it will likely be an uphill climb.


Residency Opportunities for older MD students

Older MD who are shy of 50 years old, what would be their chances of getting into a good residency in US like radiology or optho?

tantacles

Typically, programs do not discriminate based on age but use the same criteria for every applicant when extending an interview. That being said, it is hard to know how the interview would play out; ultimately, being liked is an important part of a residency interview, particularly for a competitive field, and your age may influence how you are perceived in either a positive or a negative way.


Medical School GPA requirement questions???

I'm currently a freshmen in university and I am a sociology major on the pre-med course, so far my grades are near perfect. My concern is if Medical Schools take the overall gpa from every college or from the university your currently attended. I took some dual credit classes in high school, I did fine in most of them, expect for Biology which I received a D . I took Biology again in the university I'm attending and received an A. The university I attened tranfers the credit, but it doesn't affect the gpa. My main concern is how that will be precieved when I apply for Medical School. I mean if I have all A and one D from a course I took in High School how will that look. Does it vary from each Med School or is it the same.

tantacles

Medical schools calculate the GPA using every college course you have ever taken, including those you took in high school. One D will not destroy your application as long as the rest of your grades are great.


Neurosurgery - Cardiac Surgery Combines

This question will be an odd one, but please answer the question rather than inform me of the obvious - it'd be practically impossible to accomplish. However ...

I cannot decide between Neurosurgery & Congenital Cardiac Surgery. I am invested in and love both these surgical fields and wouldn't ever want to choose between the two. Would it be technically feasible (i.e. legal / allowed by medical group bylaws) to complete a Neurosurgery residency after medical school. And then complete a 3 year cardiothoracic surgery and 1 year congenital heart surgery fellowships after this to qualify as a Congenital Cardiac Surgeon and a Neurosurgeon. Between the two could be a few years of General Surgery training; but my question is whether it would be technically allowed for a hospital to allow me to enter a 3 year CT surgery fellowship after a Neurosurgery residency without the full 5 year General Surgery residency. An idea would be a plastic surgery fellowship, which you are allowed to complete after Neurosurgery. Would this be considered enough non-neuro surgery training to be allowed (not likely) to enter a CT fellowship.

Note: I will be completing an MD-PhD program, so I would have research experience on top of that in the Neurosurgery residency. Also, I know this question is purely is it possible and not feasible, but would getting a med degree from Harvard / Columbia for example and a top Neuro residency be an advantage?

tantacles

Legally, you would probably be able to do this. Your medical license allows you to practice any kind of surgery or medicine, regardless of the training you've had (although you would likely not be hired at a hospital to do general surgery after doing neurosurgery, for example, because it opens the hospital to malpractice suits.). There should be no problem with you doing these residencies from a legal standpoint. The big question that needs to be answered is not one of legality; it's hard to know if a program would want someone who had done a neurosurgery residency given that they probably won't have had much experience with thoracic or cardiac surgeries during residency; after the first year of residency in neurosurgery, you do almost all of your months on-service. I would check with specific programs to see if that's something that's feasible.


Going into Match without a COMLEX PE score, what are options when I don't match?

I am taking the COMLEX PE in February and it will be too late for me to report the score for programs to rank me. I took it late because of financial constraints, although I know the reason really doesn't matter. I was told I needed it to graduate and did not find out about the requirement for the PE to be considered for ranking until recently. Realizing that I will likely not match into any program, will I be able to scramble for a spot during the AOA match without a PE score. Also, if I am unable to get a spot what are options for me in the interim before applying next year? I have heard research, MPH, and extending graduation are options but is there anything else I can do? The rest of my academic records has no red flags. There are no research opportunities at my school, where else can I look?

tantacles

Those three options are your only ones. If you extend graduation, you can still do research or an MPH.


ENT vs Plastic Surgery

I cant decide between the two, are there any major pros/cons that I should consider? Anyone have insights into how they chose one over the other?

tantacles

Both have very different scopes. I think the best thing to do is to pick the most routine procedure in each field and think about whether you can handle doing that procedure over and over. In ENT, for example, a tracheostomy is extremely common. If that procedure bothers you, the field is not for you.


retake mcat?

Mcat 505, overall GPA 3.66, science 3.7, worked as medical assistant 3 years, state school graduate.

tantacles

It would be worthwhile for you to retake the MCAT. An MCAT of 505 translates to a 29 on the old test. You likely have a good chance of getting into medical school with this score, but you could definitely increase your chances by retaking. However, beware: Only retake if you have more time to study. If you do worse, it will be catastrophic for your application.


Does anyone know of any medical schools that are known to give out full scholarships for people whose income is below 50 thousand?

tantacles

Many schools offer financial aid, even full scholarships, that are based on merit or financial need. I would suggest that you contact individual schools you are interested in if you wish to find out more about financial aid. Keep in mind that medical school is incredibly competitive, and you must first be accepted in order to receive aid.


Repeating the MCAT

I graduated from the University of Toronto with an unimpressive GPA (3.13). I wrote the MCAT while taking a course overload, with virtually no preparation, and scored a 508 (66th percentile). I know that I am capable of a much much better score. I fared quite well on the Psychology & Sociology section (98th percentile), CARS was okay (88th percentile) but the science-based sections were extremely poor. I know that was because of lack of preparation and that I can score at least in the 95th percentile if I take the test again. Is it worth it to take the test again before applying to medical schools? I would ideally like to get into a top tier medical school, even though that seems quite impossible with my current stats. So my question is, if I were to score in the 95th percentile range, would my bad first attempt overshadow it? Would it even be worth it to retake the test?

Pathdocmd

Without knowing anything else about your application, at this point it is your GPA that will hold you back the most. You will not get that far with a overall 3.13. A 508 is OK, but even with a 516 your GPA will be your biggest hurdle.


Vascular medicine fellowship, anyone know some details about this?

I'm a IM resident, has been thinking about apply vascular medicine, the problem is it's not ACGME certified and only 5-6 programs are accepting IM resident. Is it really competitive? How is the job market after graduation? Not much information online. Any details or commends are highly appreciated. Thanks!

tantacles

Given that there is no information available publicly about this fellowship, it is difficult to say how competitive it is. I would suggest you reach out to programs specifically and find out what kind of experiences and residents they are looking for.


Can I practice as a PA without course after usmle

Hello, I am international medical graduate, worked 5 yrs in India as physicians, can I work as Physicians assistant directly without doing PA course in some states ,what I have other options, I am very depressed please help me out!

casedentalmed

Hello, I think there are board examinations for PA's as well, so you are not allowed in the US to work as a PA without knowing the state-specific regulations and complying with them.


Urology Match

Curious about my chances in the urology match, and I'm not sure if I'm just being overanxious:

Current MS3:

Step 1: 260

MS3 Clerkships (H/HP/P):

Medicine, Primary Care, Anesthesia: Honors

Surgery: Pass - This is what I'm worried about, obviously. I worked so ridiculously hard and got amazing comments from both of my clerkship site directors, however one of them gave out blanket 70s to everyone that rotated at his site, and the other one gives a max grade of 80, which I got, so I needed to score in the 95th percentile to HP and Honors was already out of reach based on our grading system. Unfortunately, at the end of the day, PDs will see the Pass out of all of that.

OB/GYN/Peds/Neuro/Psych: Pending/haven't done yet

Currently involved in a research project, VP of my school, likely will be elected president in March, involved in other extracurricular projects. Our Urology chairman and I have a fantastic relationship - known him since prior to medical school, however I know him to be very optimistic with people about their match chances and just want another opinion regarding my chances.

Thank you!

tantacles

It's extremely hard to say what your chances are without knowing your other grades. Your P in surgery will definitely hurt you, but only a specialist in the field can tell you your actual chances. Your Step 1 score certainly will not hold you back, and t's very likely that you will be able to match, but it's simply uncertain where you'll be able to match. The reason for your pass won't matter much to PDs who have to sort through a tremendous number of applications.


Too late to consider plastic surgery?

I'm a 3rd year who always thought I wanted to go into CT surgery. I even took a year off to do CV research but after working with the cardiology patients I did not love CT surgery as much as I thought. Lately I have become interested in plastic surgery after my ENT rotation, but wondering if it's too late to pursue plastics, most people I know who go into plastics know beginning of med school they want plastics. I only recent have become interested. And now it's more than half way through 3rd year, and I have my plastics rotation set up early spring.

I have some honors first 2 years of med school, 250 step 1, honors in surg, med, ob/gyn, peds, and FM. I have 5 pubs but they are all in CV research, I know plastics is super competitive so should I look into plastics research or just apply to gen surg and consider some other specialities? Or should I just directly email the chair at my school and set up a meeting to see? Not really sure what to do...any advice would be appreciated.

tantacles

If you truly want to do plastic surgery, I would strongly suggest that you either take another year to do plastic surgery research as you have none or apply to general surgery and seek out a fellowship after residency. It seems as if it would be possible for you to match into a plastics program without taking time off, but it would be wise to take the time you need to get into the program you want.


FM sub-I or IM elective For IMG

I am an international student who intends to apply for IM residency. I have received MSKCC elective at oncology department. However, UCSD also offered a sub-I rotation at FM in the same module dates. Both hospitals can not change my assigned period of time, so which rotation should I go to?

An attending told me that in general, if you are applying for IM in the US, you want IM and IM subspecialty rotations. My AMG friend also revealed that for American students the conventional advice is to do elective instead of sub-I because the responsibilities are much higher on sub-I and you will be compared to students from UCSD who will be familiar with the system and people.

But for international grads, perhaps it'd be better to do a sub-I and prove without doubt that I can handle intern responsibilities. I have seen some comments online say sub-I can bring really strong LOR, and it is also very rare for IMG to get a sub-I. Basically FM sub-I at UCSD takes care of medicine ward plus postpartum care unit. A UCSD FM sub-I is equivalent to IM a sub-I in terms of what I do, but again I'm not positive how it would be perceived by residency programs. Since an upperclassman told me that some IM programs ask for as least 2 LORs from IM, I am quite concerned about the category of LOR displayed on ERAS.

In addition, another pros for elective at MSKCC would be the location. IM programs in NY accept lots of IMG annually. Probably it will be great to establish some connection at this point.

Thank you for your attention. Happy Thanksgiving!

tantacles

I would strongly suggest that you do the elective; you are planning to go into internal medicine, and that means that you want recommendations from internal medicine faculty in order to get into internal medicine residency. Also, on a sub-internship, any flaws you have will be magnified 20-fold because of the increased workload.


amc vs usmle vs plab

hi, i am a med student from india. currently i have completed almost 80% of my internship and i am confused about further pursuing residency plans. which is the better option and more img friendly; usmle , plab or amc exams?

tantacles

If you plan to pursue residency in the United States, you should take all three USMLE exams as most residency programs will require them for matriculation.


regarding the co authorship

Actually I am going to be a second first author, with equal contributions to the first-first author . But that full work will be my PhD thesis also, Does being a second co-first author on my thesis work is possible to under estimate my cntribution or it will create any consequences in future. Am I just a glorified second author on my phD thesis work

Neuronix

Greetings,

The requirements to have a solid PhD thesis are not set in stone. If your thesis committee agrees to let you graduate with your thesis, a shared first author publication, and any other achievements, then you are done. Some people may think that one first author publication (shared or not) is not suitable for a PhD. But again, there is no standard. Most people won't notice.

When it comes to shared authorship papers, I typically see people listing themselves on their CV as the first author. I don't think anyone would fault you for that. If this is a high impact factor journal you're publishing to, that makes it all the more likely that this will likely be overlooked in the future.

The question becomes, what are your long-term goals? Are you PhD only or MD/PhD? Is your goal a more clinical career or a mostly basic science career?

No matter the answer above, I don't think this issue will be a major setback for you. If you are PhD only, it may make sense to move to a post-doc now versus staying in the lab extra time to generate extra publications. If you are MD/PhD, I think it makes sense to move on if you've been in your thesis lab a suitable amount of time (4 years or so) and plan to either transition to a more clinical/translational pathway or plan to spend more time doing basic science. Even if you are a star in your PhD, needing more time later in your career to establish independence would likely be the case.


What factors should I truly consider when choosing between multiple medical schools that I have been accepted to?

I have been accepted into multiple medical schools, and I do not know what factors to consider. Should I base my decision mainly on board pass rates, and residency placement rates, or do I let considerations such as cost, living area, etc play a role?

tantacles

All of these factors are important. Residency placement and board pass rate are certainly important as is location; you want to be happy during medical school. Two other things to consider are:

1. Cost of medical school - you will be paying off loans for a while after medical school. a cost difference of $10,000 isn't much, but $100,000? Big difference in loan volume.

2. Culture of the medical school - some medical schools are more supportive than others.

Ultimately, it comes down to what YOU want. What is most important to you? That should guide your decision.


Cardiac Surgery vs. Sports Medicine

What is the general post residency/fellowship lifestyle (free time, vacation days, on call schedule) of a sports medicine surgeon and a cardiac surgeon specializing in aortic and aortic valve surgery? From what I've read already both have similar pay and training time (assuming you do the integrated thoracic surgery residency), so those are non-factors, and I have equal passion for both specialties.

tantacles

As a surgical sub specialist, your practice type and personal preferences will determine your schedule. Both lifestyles will be difficult, but as an attending, you could choose to be in a private practice and operate only one day per week or you could work in an academic center and be busy all the time. Choosing between these fields, I would consider your personal preferences more than anything else.


Usmle step 1 score of 238

I am an IMG .I recently took usmle step1 in September 2016. My scores are out 238. Wanted to know how good a score is this.

tantacles

238 is above the national average for Step 1, which is about 228-229. Since the SD is about 18, a 238 is about the 70th percentile or so.


prospective of international student who is doing his undergrad in US to go into med school?

I am an international student, a freshman in one of US colleges. I aspire to go into US med school. But just being international student makes the possibility so slim. Any help or advice or alternatives other than US med schools would be appreciated.

Pathdocmd

You will have to inquire at each medical school and this information is usually on their webpage.


No matter how much I study, I end up getting just PASS grades at my medical school and it has significantly increased my stress level. Is it possible for me to take a time off from my medical school and continue the program? I am planning to take at least one year off my school. Appreciate your comments

I only get PASS grade no matter how hard I study - I do not know who I can talk to get any feedback on my performance. My stress level has gone up due to this poor result.

casedentalmed

Most medical schools should have a designated faculty mentor or a dean/director of student services with whom you can share your concerns and ask about the leave of absence policy (if it is not articulated in your student manual). I hope it is clear who that person is in your program.


Internal Medicine LORs

I did a 4 week away rotation and got 2 strong letters from that; is it ok to include both? My other two letters would be the dept letter and a 3rd year medicine letter. My letter from my subI at my home institution would not be as strong. Is it generally a bad idea to use two letters from the same rotation?

tantacles

If you have two very strong letters from a rotation, it is completely acceptable to use both.


Residency comparsion between Canada and U.S

Im a graduate of a middle eastern med school. I am also canadian.. I cant really decide whether to do residency in the us or in canada. what are the pros and cons of each?

tantacles

Residency programs in both the United States in Canada will prepare you to practice in both places. Programs vary widely, however, so I would urge you to seek out information about individual programs; it is hard to generalize given the myriad specialties and programs available.


When applying to an internal medicine residency program, should I include my Departmental Chair letter of recommendation even if the program does not explicitly request it?

Residency application process. Some programs will ask for 3 letters, "including the chair letter," while some will just ask for 3 letters without any further instruction.

tantacles

I would include a chair letter with every application to an internal medicine program; these letters are valuable to residency programs.


New Navy uniforms for HPSP

I'm in the Navy HPSP and a month into my first year of med school. I saw the Navy is switching from aquaflague to NWU Type III, but I've seen conflicting reports on when this will start. If I'm planning on doing officer development next summer (2017) any idea on if I'll get this new uniform or if I'll have to buy the old one and then get the new one right after? Thanks in advance!

casedentalmed

I hope you got clarification in advance of your officer development. Sorry we couldn't really answer you.


Requesting an interview

My number one school has been giving out interviews and I haven't been contacted. That being said, I am going to an interview for a school in the same state that is my second choice. I read online that my first choice school has interviews lasting until February, but I would have to make a decision for the second school by Dec 15. I heard that you could request an interview from a school if they hadn't offered but I was wondering if that would alter how they see you, if they judge you, etc. My main question is how do you request an interview and if this is a good idea?

My sGPA is 3.38, overall 3.43 (with grade forgiveness 3.53/3.54) and MCAT 508.

My first pick school says their entering class stats were GPA 3.7-3.75 and average MCAT 506. I am applying out of state for this allopathic school.

I've had a lot of medical experience as a CNA (~3 years), worked as a volunteer in a free medical clinic (2 years), shadowed a DO, and held a job throughout college waitressing. Any help would be greatly appreciated.

Pathdocmd

We (Wake Forest) get requests like this from people interviewing at Duke or UNC. Many are already on a list to be invited anyway and so when they send a request we try to accommodate. I cannot speak for other medical schools.


How do I approach going into the match for 2017 to switch specialities after a successful 2015 match.

I am a PGY-2 emergency medicine resident and want to switch to radiology. I have talked to my Program Director who is supportive. I am looking at exciting open residencies, but am also considering going back into the match (March 2017). How do I go about getting an ERAS token, Dean's letter, etc. Do I work with my medical school or my current program director. Basically, what's the process to go back into the match.

ExpertsManager

Unfortunately, we do not have an expert who can answer this specific question. However we encourage you to ask this question in the General Residency forum ( http://forums.studentdoctor.net/forums/general-residency-issues.49/ ), where there may be others who have experienced this situation. If you would prefer to remain completely anonymous, you can check out the Confidential Consult Forum ( http://forums.studentdoctor.net/forums/confidential-consult.268/ ).


how to start residency in UK as an IMG+ EU citizen who just got Full registration with the GMC

Hi, I m a citizen of france and an IMG. I have completed my 2 years internship and got the full registration with the GMC in UK. I need to know please how is the process to get into the specialty program. Thx

ExpertsManager

Hello,

Unfortunately our Expert's expertise only pertains to North American medical school and graduate medical education. However, we do encourage you to ask your question in our International Forums ( http://forums.studentdoctor.net/forums/general-international-discussion.85/ ).


nbme score

i did nbme 17 online and scored 241 , what shall i do next to confirm my mark

tantacles

I would suggest taking other NBME practice exams as they contain actual questions from past exams, and doing more practice exams will give you a better idea of what score you will get.


Foreign physician with green card apply U.S. medical school

Hi, I was a practicing physician in China. Since my family did immigration, I came to U.S.

Cons: 1. I have graduated from medical school about 10 years. It would be very challenge for me to match a U.S. residency program. 2. My bachelor degree was earned in China. 3. I am in the middle of 30s.

Pro: 1. I have a Master degree in the major of Biomedical Science in U.S. (GPA:3.71) 2. I have research experience with scientific publications as a first author and co-author. 3. I have green card. 4. I had clinical experience.

Entering medical school in U.S. is a serious decision for me and my family. Now, I am in a MCAT studying group with American students. Meanwhile, I am doing volunteer work and looking for the opportunity of shadowing. I am wondering, as a non-traiditonal student, is there any chance I can get into us medical school? How to improve my application? Thank you!

casedentalmed

Many medical schools offer programs for international/foreign medical graduates which would circumvent your need to do the entire MD/DO curriculum. You may want to check each admissions officer at your schools of interest about that since you may not need to do the MCAT.


Low cGPA (3.0), MCAT 484 chances of USA or CANADIAN Med School?

HI, Just wanted to get your opinion. I have a low CGPA of 3.0, and around the same if not lower for science. I did my mcat for the first time without using any external help from a program and got a score of 485. I know I wanted to go to med school, but i don't think ill get in applying to a Canadian school or even american with these numbers. Should I go to a Caribbean school (Ross or Saba) or finish my masters for 2 years aim for a high mark and re-write the mcat and apply again for a Canadian. Keeping in mind i'll finish my masters at the age of 25 and then start medical school which mean i'd graduate at age 29/30 and start looking for residencies. OR having an opportunity I can go straight to Ross or Saba, because whats the guarantee that even after completing my masters I'll get into a Canadian school? That would just mean that I "wasted" 2 years doing a degree and have to start in the Caribbean, something I could've done 2 years prior. IS it really worth waiting to re-try in two years or should one just go straight to the Caribbean.

tantacles

With this gpa and mcat, you are unlikely to be accepted into US allopathic or osteopathic medical schools without doing a formal post-bacc or special masters program. I would not recommend that anyone attend a Caribbean medical school; even for the best students graduating from Caribbean medical schools, the mere attendance of a Caribbean school makes many residency programs turn away your application after your final year of medical school, meaning Caribbean graduates are much less likely to match than their US allopathic or osteopathic counterparts. In addition, their model often requires them to cut certain students out of the program based on academic performance with little chance of redemption, thus artificially increasing their match rate and making them seem like a much easier and more accessible option than they actually are when compared to other routes.

Thus, my suggestion is that you take time off to improve your undergraduate gpa with a formal post bacc program, repeat your mcat, and wait to apply, or that you reevaluate your choice of medicine as a career path and choose an alternate route to take for your life.


What is the general consensus about foreign rotations as an elective during fourth year in medical school?

I am a US allopathic student, and am wondering how foreign rotations taken during fourth year are perceived by residency programs. I am interested in a competitive surgical specialty, and need to make the most out of the elective options that I have fourth year. That being said, are foreign rotations perceived as a waste of time? Should I save my electives for away rotations here in the US? I really like to travel, and see a foreign rotation as a way to learn about different medical systems, gain cultural competency, and possibly gain experience with diseases that are not common in the US. I am having trouble finding information about how a foreign rotation will impact my application, and have even received conflicting information about this topic by administrators at my school. Any additional insight will be extremely helpful!

tantacles

Generally, for US allopathic students, the only rotations that have any relevance in terms of evaluating applicants are third year clerkships and 4th year subinternships. Other rotations have minimal effect on the residency application, and most, particularly if taken after eras is submitted, do not appear on the application in any case. Thus, do whatever rotations in the fourth year that interest you and that you feel are relevant.


Does med-school effect your personal life,my father is worried that i won't have time to get married or have time for kids .

tantacles

Balance is difficult in many fields, and medicine is no exception. That being said, many medical professionals raise children and get married, so there is no reason to believe you would not be able to accomplish those goals if they were a priority for you.


To postpone match or not?

So I am currently in a confusing situation and would appreciate any input. I am currently a MS3 from SGU and am finishing my last core rotation in surgery. I want to apply to either general surgery or IM and am currently trying to decide what the best course of action for me would be. I have a 128 on my Step 1, 3.7 GPA and all A's in my cores (except a B in OBGYN). I am trying to figure out if it would be better for me to postpone my residency match to 2018 or to try to cram in my surgery shelf, CS and CK exams all within the next month and a half. If I end up trying to match for 2017 I will have mediocre Step 2 CK scores and LORs, but if I push match to 2018 I am confident I can score a high mark on the CK and obtain excellent LORs, plus I would do research throughout 4th year and the next year and hopefully get publications. So how bad does it really look to residency programs if you postpone match for a year (I would still be graduating on time)? Also these options are based upon my assumption that I must have the CK complete by the end of August. Is it a very bad idea for me to take the CK at the end of September and have a full month of free time to study? I appreciate any advice I can get from people who have recently gone through the match or know the process well as I have been getting very conflicting advice from my schools advisors and other people I have been talking to. Thanks!

tantacles

If you were to postpone the match for a year, it would be better for you to postpone and NOT graduate on time, taking the extra time to do research. Schools prefer students who have not yet left medical school as these students tend to be closer to medicine. If you choose to do research, it will also strengthen your application, which is key for an international graduate.


Do I make the cut?

I'm 25 years old, white male, Born in U.S Year in school: Graduated December 2013 Country/state of residence: Currently SC moving to PA/NY for new job hopefully (establish residency) Major: BS Biochemistry, BA Economics Schools to which you are applying: Weill Medical, University of Pittsburgh, Temple, SUNY Upstate, SUNY Downstate, **********Just beginning my list, Open to suggestions for schools that are good fit in either PA or NY. From what I understand my list should be ~15 schools********** Cumulative GPA: 3.7 Science GPA: 3.5 MCAT Scores: 513 (127,129,129,128) Research: Virtually none outside of a few days of some very interesting neuromonitoring studies on innovative pedicle screws being tested on pigs, which I don't think I'm supposed to talk about... All of this work was done through my current job (Intraoperative neuromonitoring technologist) Extracurricular activities: Admittedly weak here as well: Unfortunately used to be partying :'( but also intramural/pick up sports - Soccer, Basketball, Tennis. Nothing too impressive here... Do board games count???? I play a mean game of settlers of catan! Employment history: Intraoperative neuromonitoring technologist starting right after college - present (~3 years) My job is a little obscure so I'll try to briefly explain it: Basically I monitor the integrity of the nervous system during basic spinal fusions, as well as more advanced surgical procedures including removal of intramedullary tumors and craniotomies. This requires efficient communication with patients, surgeons, OR personnel, and online neurologists/professional overseers. Basically I'm a specialized tech that knows where to stick needles during surgeries to make sure nothing too traumatic happened to the spinal cord during the procedure. In college I was a handyman during summers and coordinated a group of guys in various oddjobs (painting, landscaping, etc.) in our entrapreneurish group called "four strong men" - On a side note, I think I'll keep this on my resume forever, I get positive feedback all the time and people love the name, even if it is a little childish. I had various non-serious jobs since I was 14 up through college, not sure if these little hustles of mine are even worth mentioning... Immediate family members in medicine? No Specialty of interest: Family Medicine, Psychiatry, Neurology, Mostly unsure, Definitely NOT surgery - Would like to learn a lot more in school! Shadowing experience: Never Shadowed, though I do work with surgeons/Anesthesiologists/Neurologists/Nurses all the time as I go in to monitor cases daily. Graduate degrees: No Interest in rural health: Yes, but right now I need to focus on work and applications, gotta pay those bills!

tantacles

Based on your experiences and gpa, you have a good chance of getting a medical school acceptance from a program that values clinical care and volunteerism more than it values research. Given your lack of research experience and the fact that your GPA and MCAT are good but not stellar, I would recommend either taking a year or two off to address your research deficiency and/or improve your gpa. If this is not possible, I would recommend applying to 30 schools total and broadening your scope to include programs in more programs in areas that are not New York or Philadelphia as these programs tend to be quite competitive. In medical school, you must go where you get in, and you must get in somewhere to become a physician, so apply to many varied programs and choose your final destination wisely.


Step 1 Score - 251, Step 2 - 248, 6 publications,founder dermatology interest and history of med group what are chances of receiving interviews in dermatology

Am I wasting my time pursuing dermatology with the below info.

Step 1 - 250, Step 2 - 248 really bummed about the 248. Scoring in 260's on practice. A's in all rotations except IM - B Stellar reviews on all rotations 5 publications - 3 first author, contributed to a few chapters in a new dermatology textbook, asst editor on derm textbook Co-founded two clubs in med school - Dermatology Interest and History of Medicine Volunteered Do you think personal statement would help? Professional actor as child - performed on Broadway - will this help? Applying for AOA - 4th year

tantacles

Based on your experiences, step 1 score, and clinical grades, you have an excellent chance of receiving interviews in dermatology. Depending on your school's us news and world report rank and general reputation, it is hard to say if you will receive interviews at top academic programs, however. I would recommend speaking to an adviser at your school in this field or finding a adviser in this field on an away rotation who can better speak to your chances.


I was not accepted to an MD program; 2 application cycles.

I am a non-traditional pre-medical student; I am currently 29. During both of my application years I received interviews at several medical schools and was wait-listed both years at multiple schools - but not accepted anywhere. I scored in the 93.7th percentile on my MCAT and my gpa is 3.93. I taught for 2 years in an undeserved elementary school. I spent more than 4000 hours working in a group home with developmentally disabled and mentally retarded adults. I spent 9 months working in a chemical dependency clinic and more than 100 hours working in research (which resulted in a publication). I have a whole smattering of charity and volunteer work on my application -all supported by letters of recommendation. My application and volunteer work were built around the advice and guidance of the pre-medical adviser at my school who also wrote me a "Board" letter. My application and essays were reviewed and critically analyzed by two different people with masters degrees and their feedback was incorporated into my application. I applied to schools both above and below my grade range, both in-state and out of state 29 schools in all over the two years. My interviews skills aren't bad or terrible and there is nothing outwardly wrong with me; no speech impediments or disfigurements. While my opinions are generally conservative, I was as diplomatic and honest during my interviews as possible and kept clear of any ideas or talk that would have suggested otherwise.

I spent the better part of 5 years busting my behind to be successful and spent upwards of 35,000 dollars on this expenditure. What makes me unqualified? I deserve an explanation as to why no one wants me - even though the sacrifices I've made to get to this point probably dwarf the lions-share of those students who have been accepted - working 90 hour weeks between school, volunteering and my regular job for the better part of the last 5 years. Why wasn't I accepted anywhere? Is it my white skin, my male genitals or my heterosexual orientation that make me unqualified to be a doctor?

casedentalmed

For any reapplicant to MD programs I would strongly urge applying to DO schools as well, no matter what your metrics are. DO schools are very competitive, and are no longer "shoo-ins" for an applicant with your numbers. One would think that with a variety of schools that you have applied to that you might get some interest, but perhaps it's a question of your timing of application or that your application doesn't really resonate with the majority of admissions committee members you have interacted with. There are a lot of applicants with lots of shine and sparkle but no depth, and there are many more applicants who feel that medical school is the "brass ring" and the trophy that validates a job well done rather than a true lifelong calling and pursuit.

If you believe you are being discriminated against in this process, I am pretty sure there are many Caucasian male heterosexuals who are getting into medical school who are willing to be part of a complex, diverse workforce to manage the health of other patients and populations. It's too difficult to say what the issues are; sometimes there are enough better candidates that you get squeezed out.

So you need to take a real strong look at your application and get it reviewed very critically. You need to figure out your application strategy and include DO programs. If you think you deserve an explanation, be disappointed because often there are no explanations. There is a lot of luck in the process, and you have to be humble and respectful enough about the process to accept the failures with the successes. You will face many more failures and imperfect explanations in your medical education.


Step 1: failed the first time (179), aced it the second time (256). What are my chances at a good surgical residency?

Due to unfortunate circumstances, I failed Step 1 the first time. I got a 179. I took it again and got a 256, which was consistent with my performance on practice test I took before getting the 179. Even though personal life events and extenuating circumstances largely contributed to failing Step 1 the first time, I am concerned that residency programs will not consider it. My medical school deans have given me very little guidance.

Do residency programs consider or give weight to a very good second Step 1 score? How much does Step 2 and 3rd year rotations factor in? I haven't not taken Step 2 yet, but I have completed two rotations (neuro and psych) and received Honors in both.

Despite the 179, I have many strong points on a future application. I have 9 published papers, am chair of multiple committees, interest groups, on executive board of the free clinic, have volunteered several hundred hours, am editor in chief of the medical school's journal, help run a and non-profit organization....

I have two good connections who said they are willing to make calls for me to programs I will apply to.

I know it is hard to anticipate these things, but what do you think my chances are of placing into a good surgery residency? Are programs in Plastics and Urology out of the question because I failed Step 1 the first time? Do you have any advice on what my next steps should be?

Thank you in advance for any advice you may have. This is something that is very troubling to me and I worry frequently that my prospects are very bleak. I have never participated in online forums like this before, but I feel that posting on SDN is my only lifeline especially as the deans at my school do not have constructive counsel.

tantacles

While your second step one score will certainly carry weight, a failure on step 1 is seen as a red flag by many programs and will disqualify you from interviewing. Competitive specialties, without tremendous mitigating circumstances, are out of the question. While it is possible for you to gain a surgery residency at an academic institution, I would think it unlikely. When you apply, I would recommend casting a broader net than most and applying to a less competitive specialty as a backup plan.


Wait-List transcript update question at a middle-tier medical school

I am currently on the wait-list (since late April) at a middle-tier medical school. I have already sent in a letter of intent, and wish to remind them that I am still highly interested. Unfortunately, the only update they still accept after the LOI is a final transcript, of which I received 3 A's and a C+ (my first of college) in a graduate-level viral pathology class. I wish to include quick updates along with the transcript when I send it, but am hesitant as I fear them seeing a C+ will wipe me off the Wait-List. Any and all comments/advice would be greatly appreciated. Thank you in advance!

Pathdocmd

I doubt sending it or not will make any difference this late in the admissions cycle. I can only speak for my med school, but we really don't take additional courses into consideration. I would contact the school and ask if they do take this into consideration. Unless your sGPA needed some serious help (and for some reason you think this has to do with your wait list status), I probably wouldn't send it.


Residency in USA

I came to know that we can apply for Residency programs with only STEP 1 score with a condition that STEP 2 score be available before joining Residency program. Is it correct? If yes, will you please provide me the list of Residency programs and the Universities in USA which accept applications with only STEP 1 score

tantacles

As you say, it is possible to apply for residency with only a Step 1 score and not a Step 2 score, though many programs will not add you to their rank list (required to match to these programs) without a passing Step 2 score by the time the rank list is finalized (February). Given that there are hundreds of residencies in the United States, I will unable to provide you with a comprehensive list, but keep in mind that most programs post requirements for their residencies on their websites, and I would suggest looking up each one to figure out its exact requirements.


Medical Examiner v. Forensic Pathologist

I've done a lot of googling, and can't seem to find a clear answer .

So.

What is the difference between a Medical Examiner and a Forensic Pathologist??? What are the differences in schooling? What are the differences in career? What are the differences in requirements and expectations? HELP ME UNDERSTAND.

Pathdocmd

What is the difference between a Medical Examiner and a Forensic Pathologist??? A Medical Examiner is employed or contracted by the state/county/city to investigate deaths in its jurisdiction. It varies from state to state on the requirements to be an medical examiner (ME). They may be a board certified forensic pathologist, an non-forensic pathologist, a non-pathologist physician, or a non-physician. A forensic pathologist is pathologist that does forensic investigations and autopsies. They may or may not be board certified in forensic pathology. In short, not all MEs are forensic pathologists and not all forensic pathologists are MEs.

What are the differences in schooling? A forensic pathologist does 4 years of college, 4 years of medical school, 4 years of pathology residency, and 1 year in a forensic pathology fellowship. The requirements vary from state to state on MEs.

What are the differences in career? An ME will investigate deaths for the government and that varies from jurisdiction to jurisdiction. Wikipedia has a nice explanation" <a href="https://en.wikipedia.org/wiki/Medical_examiner" rel="nofollow">https://en.wikipedia.org/wiki/Medical_examiner</a>

A forensic pathologist does the medical examination (e.g., autopsy, interpret toxicology results, etc) of the decedent. He or she may do forensics full time as an ME or just part time as part of a pathology practice or medical school faculty.

What are the differences in requirements and expectations? I'm not quite sure what you mean. It depends on the government jurisdiction and job title. They could be one in the same person or they may not be.


can i take classes at a community college over the summer to boost my sGPA even though i already submitted AACOMAS and AMCAS?

Pathdocmd

I can only speak for my institution, but we do not generally look at grades that not in AMCAS.


Visiting before Applying?

Is it appropriate to visit a medical school before applying? I will be entering my sophomore year of pre-med and will be up in the area of some potential schools, and would like to visit while I'm already in the area, even if it is quite early in my pre-med path. Is this considered to be too pushy and rude? Or would the initiative be viewed as a positive thing (I might want to schedule a meeting with admissions as well to discuss a non-traditional situation)? I will be particularly looking at MD/PHD programs.

Thank you!

Pathdocmd

I wouldn't say it is inappropriate, but it doesn't happen that often in my experience and it doesn't help or hurt your application when you do apply. Med school admissions offices are not like undergrad offices. If they do let you come, they have to work around their schedules and the meeting might be brief. Don't expect to see the dean or any faculty members. You may or may not get a tour. I assume that each med school has a different policy. If you have specific research interests, have your PI contact the PI at the med school.


Can I still practice medicine in the US?

I am a US resident and I am considering the option of attending a Canadian medical school instead of an american medical school. I just wanted to know if it's very complicated to attend med shook in canada and practice in the US? Would I have to take some kind of exam to get validation or some sort of program after medical school? Can I still do my residency in the US even if I attended med school in Canada? I also know that sometimes they put you a lot of "obstacles" when you come from a foreign medical school, so I just want to know how difficult it can be if I study in Canada. Thank You

tantacles

Most American residencies have no problem taking Canadian residents as they are considered American Medical Schools. It should be easier for you than a normal Canadian medical student because you will not need a visa, but you should make this clear when you apply for residency.


Residency matching

I am an international medical graduate from India interested in psychiatry residency in the U.S. I finished my medical school last week and have finished my step 1, step 2 ck and cs. I passed all on first attempt but my step 1 score is 212 and step 2 score is 211. It's a little disappointing going through the NRMP data that says that only 132 IMGs of 1384 positions get into psychiatry so my chances look pretty slim. Nevertheless I plan to apply and be positive. I have got clinical experience of 3 months in the U.S. with 2 months in both inpatient and outpatient psychiatry and 1 month in pediatrics (non-psych) and will receive letters from there. I was wondering what I should do to increase my chances to get interviews and get matched.

tantacles

Getting more clinical experience in the United States is always a great idea. In addition, doing some psychiatry research would be positive.


Late Application To Texas Medical Schools

I am a reapplicant to Texas medical schools. I applied last year with a 27 on MCAT(10PS-6(!)V-11BS), I had a 3.86 GPA. I was done with secondaries in the first week of June. I didn't apply to TCOM, because I wanted to take a second chance for MD schools.

I have done extensive research during undergrad, I have been a leader for more than 5 programs, in addition to healthcare volunteering and shadowing. I had one interview at A&M in December, and I was placed on a waitlist. I contacted admissions directors at many schools and all told me my MCAT was the major factor for rejection. I am retaking the MCAT in July, and I plan to submit TMDSAS and complete my secondary applications before my score comes out in August.

Is it too late to apply in August? My GPA after a year is a 3.92 and if I hope that MCAT will be 508-510 (I have been consistently getting higher than 510 on practice exams). Should I just wait to apply next cycle? Or take my chance to submit the application for this cycle?

casedentalmed

It's not too late. While I cannot speak for how the schools will treat your new MCAT scores, I will encourage you to apply to DO schools. Give yourself a chance to cast a wider net. You gave yourself a second chance already for MD schools, so if you really are serious about becoming a physician and not so much about the letters after your name, send an AACOMAS application.


PTAL extension after expiring the first USMLE exam score!!! what i should do?

Hi, could any one help me out , my PTAL will be expired when my step1 score expire after 10 years if some one is not getting residency in California . any solution to to keep valid PTAL without Re-writing step1 exam again !!! I am old graduate, very good experience in Family medicine and have complete training in OBGYN from back home . Since last 4 yrs applying for FM residency but no good luck.

aProgDirector

Reviewing the PTAL requirements, I do not see any expiration of USMLE scores. If your first step pass is 10 years ago, then you may be unable to take Step 3 without retaking earlier steps -- but this only becomes an issue when you actually apply for a license. For the PTAL, it looks like you can just apply to renew it.

However, if you took the USMLE steps 10 years ago and have tried to get a spot for 4 years without success, you simply may not be able to get a spot. In addition, I'd consider looking at other states than California, as California tends to be more competitive. Your best move is to forget the PTAL and apply very broadly to FM programs in less competitive locations.

Regardless, if your steps are 10+ years old, you will ultimately likely need to retake them (assuming you have not taken and passed Step 3). If you have passed all Steps, then there is nothing to worry about.


I just recieved my undergrduate degree. Which route should I take in improving my gpa?

I just received my undergraduate degree. My current GPA is a 3.29c 3.13s, with 144 total hours (some of my credits from research didn't go towards my degree, as well as re-taking two classes). My plan is to re-take the MCAT (503 on the first attempt) in hopes that I can achieve a high enough score to be accepted by SMPs.

If I wanted to use this coming year to improve my undergraduate gpa, should I re-take classes, sign up for upper elective classes that won't go towards a degree, or apply for a second degree? Is it even worth it for me to take more classes with 144 hours earned already and trying to take the SMP route?

Any advice would be greatly appreciated.

casedentalmed

The postbac master's program is pretty much your main option at this point. Taking more undergraduate courses won't make as much of a dent in your overall and science GPA since you already have over 100 hours on the record. Retaking the MCAT may be a bit premature until you already have the SMP acceptance. I wouldn't really waste my money to take additional undergraduate courses unless you really haven't taken the upper-level recommended courses (genetics, biochemistry, cell biology, physiology, histology, etc.).


What are my chances of Acceptance to US MD schools?

Undergraduate Major: Biochemistry and Molecular Biology State of Redidency: New Jersey Overall GPA: 3.79 or if I take a summer course can bump up to 3.80 BCMP GPA:3.70 MCAT: Taking in June Research experiences: Three different Projects (Three presentations at Symposiums) Author on abstracts Symposiums: 1. American Chemical Society Symposium

                              2.Biochemistry and Molecular Biology Symposium
                              3. NAMS Research Symposium

Volunteer- AtlantiCare Regional Medical Center ~180 hours

                        AtlantiCare Cancer Center~20 hrs
                        AtlantiCare Healthplex~48 hrs
                        Nursing home ~25 hrs

Extra curricular activities: Alpha Lambda Delta Honor Society, Golden key Honor Society, pre health professionals club, circle K , volunteer tutor at Chemistry Society of School, Teaching Assistant for Chem Lab Physician Shadowing: Trauma Surgeon `21 hours Looking for an internal medicine and an oncologist to shadow Participating in Premedical Urban Leaders Summer Enrichment (PULSE) 2016 (at Cooper Medical school of Rowan University)

casedentalmed

It's too hard to guess for "all" MD programs since every school will look more favorably about your passion to pursue medicine and the source for that passion. That said, you have as good a shot as any if your MCAT score is acceptable, your recommendations help you, and your interview is solid. To that end, you need to see what schools where you best match your interests with what those schools can offer you. I'm not sure if you are inclined towards more clinic practice, especially in mostly urban backgrounds, or towards more research (as shown by your chemistry activities), or perhaps both.


If I defer my admission at a DO school through a post-bac program, can I still apply to other DO/MD schools during the year off?

I gained my admission to a DO school through their post-bac program. However, I want to apply to other DO schools since I do not like my current school after a year attending there. I want to defer my admission at my current DO school. Can I apply to other DO schools during the year off? Do DO schools have a connection where they know who already get accepted somewhere else?

tantacles

No. If you defer your admission, it is assumed that you will not apply to other schools. If you apply to other DO schools, you will forfeit your acceptance.

casedentalmed

It is fine to apply to other DO (or MD) programs but you should in good conscience decline your guaranteed seat. Don't use a deferral as a safety net, especially if you know you won't be happy there as a student.


Having a family in medical school or OB/GYN residency with military spouse.

I am looking for some advice on how starting a family during 3rd or 4th year of medical school affects residency. I am looking to go into OB/GYN. I also have a husband that is active duty, so I have a few things to consider and hope they will all align in the end. Please shed some light on this if you can. Thank you.

tantacles

It is hard to say how starting a family will impact you as the factors involved are highly variable based on the individual. That being said, many OB/Gyn residents do, in fact, have children during residency, so this is something that is understood within the field. However, particularly with a husband in active duty military, you will almost definitely require the help of a childcare specialist to help you fulfill all of your obligations. Keep in mind that depending on your combined income and budget, this may stretch you thin. That being said, again, many OB/gyn residents begin families during residency, so it will be your ability to manage your time that will ultimately determine whether or not you will be able to successfully begin a family during residency, whatever that means for you.


I have done my B.Sc. and M.Sc. in biotechnology from India, I am a permanent resident of the USA now, am I eligible for a MD program in the USA; or I have to have a premed degree?

Hello

I have done my Bachelor and Master of Science in Biotechnology that includes all the discipline of Biology except psychology. I had calculus, statistics and probability in my mathematics curriculum in my B.Sc. and M.Sc. After that I have got my PhD in Structural Biology; all from India.

After that I immigrated to the USA, and have worked as a postdoctoral fellow here. I am a permanent resident of the USA. I have published research papers in international journals, both in India and in the USA.

Am I eligible to apply for a MD program in the USA with my educational qualifications, or I have to have premed degree to be eligible to be a doctor? Do I have to have GRE/TOEFL test score, along with MCAT score for a MD application process?

Also do I have to have the official transcripts of my degrees, or the Marks sheet/ University Degree is enough for the application process.

Thanks for your help, I would appreciate your prompt response.

Sincerely Pratibha


tantacles

In order to apply for an MD, you generally have to have completed the following courses, and most programs require that you have completed all of them at a University in the United States.

1 year of general (inorganic) chemistry with laboratory component 1 year of biology with a lab component 1 year of organic chemistry with a lab component 1 year of physics with a lab component 2 college level mathematics courses (varies) 2 college level English courses

Many schools have other requirements as well, and those can be found on the schools' individual websites. Some schools require specific mathematics courses, humanities courses, or social science and psychology courses.

AMCAS will require the official transcript from all Universities you have attended, and they must be sent to AMCAS directly from the University you attended.

The MCAT is generally the only standardized exam required for admission to medical school.


Is anesthesiologist rising and will it still be a good field by 2029?

tantacles

This is a very difficult question to answer as predicting the future is impossible. Anesthesiology will no doubt still exist, but most analysts project that compensation for all fields, with the possible exception of primary care, are currently falling due to new regulations by the Affordable Care Act. However, anesthesiologists will also be needed, and they will most likely at least continue to be compensated fairly for their work.


BPD and residency prospects

I'm currently being assessed for borderline personality disorder. I have gone undiagnosed for much of medical school and recently it has affected my performance. I have taken a leave to get it addressed with individual/group therapy. Assuming I get better and return and perform well, would it still be possible to get a residency spot or should I start looking for alternative career options?

tantacles

Assuming your disorder is under control, there will be no need to search for a new field; many people with controlled mental illness do well in residency and beyond.


June MCAT

I took the MCAT in 2014 and received a 31. I do not feel this score is representative of me, so I signed up to take the new MCAT on June 2nd. I am also intending on applying this cycle. My questions are: 1. If I apply to schools before my new score is released, will the schools be aware that I have a new score coming? Will they only see my old score and judge me on that?

2. Will waiting until my score is released in early July put me at a disadvantage in applying to schools that are rolling? I know for some schools it is really advantageous to apply as early as possible.

Pathdocmd

On the AMCAS application you can indicate that you are taking the MCAT again. At our school (Wake Forest), checking that puts your application on hold until we get the new MCAT. I cannot speak how other med schools do it. For us, if you do not indicate that you are taking the MCAT again, we will use the first score.

It is always best to get your applicant in as soon as you can, but early July would not make a huge difference in my opinion. If you had a significant better MCAT, then it would be worth it (31 isn't bad BTW). That is a choice only you can make based on how well you think you will do and the strength the rest of you application.


Should I be a doctor for the nervous system?

Do you think I can be a neurologist. I have staright A's ever since I was in 7th grade to present. I really enjoy learning new things about the brain. I recently had my mom go to the hospital because she could not walk or talk right after having a severe headache. I kept researching what could have occurred to her since I wasn't getting the answers I wanted. The closest diagnose I came up to was meningitis. The doctors say she had antibodies attack her cerebellum. Well now the antibodies are "bugging" her entire brain and she seems to be having symptoms of psychosis. Getting to the point, will I be able to be a doctor with the desire and intellect I have right now? By the way I would be a first generation if I went to medical school and I am an immigrant in the US. :( no hate please

casedentalmed

I hope your mother is okay with her illness. That said, your career decision should not be driven only by your emotions. Neurology is a very complex discipline of medicine that requires a lot of intellectual thinking and emotional intelligence in working with others who have mental illnesses and physical disabilities. You need to be able to handle working with this patient group (of all ages and backgrounds) for 10-20-30 years or so. There are also many nurses and occupational therapists who also work with patients with nervous system deficits who you should also talk to about what it is like to work with this group of patients.


Going to Finally Attend Medical School - Negativity about the lives of physicians today

I'm 24 (male) and have the opportunity to start medical school this fall. I have talked to doctors in person and doctors in online blogs - sometimes they talk about how they would never ever get into medicine. They are getting less money, school debts are very high, the high salaries don't come in until your well into your 30s. In a world where I always aspired to become a doctor I avoided these negative thoughts. But, now I've been entertaining these thoughts. It does not seem practical to study medicine at all. Especially when I want to have a family, help my kids with homework, have hobbies like fishing etc. It seems criminal the amount of money one must give to be one of the most helpful, altruistic members of society. My personality makes me determined and stubborn - I will complete complex and difficult problems and persevere through difficult situations. This can be bad when you persevere through an avoidable quagmire of problems. I'm worried about what I may get myself into - a lifetime of problems? Even the doctors I shadowed told me not to become a doctor.

This is why I may consider PA because I could do medicine with a high enough salary without nearly as much debt and in a fraction of the time.

This is what I would like to know: I hear a lot of negativity, is it true? Is it no longer worth while?

casedentalmed

You should have done this homework before applying to medical school. It is reasonable to have such doubts as a practitioner, but you should have found a way to talk about these doubts to the physicians you shadowed or PA's you should have talked to. Do you really want to spend another year taking a new standardized test, doing research on PA programs, and interviewing again?

Do I think the negativity is overblown? Maybe. It's more attractive to write on a controversial topic than a story about "how happy I am" for blogs, but I'm sure there are many physicians who can't think of doing anything else but being a physician.


DEA NUMBER DISQUALIFICATIONS?

My friend was trying to become a police officer when he was 21. He is now 26, when he was 14 he did cocaine twice and when he was 17 he did cocaine once. He smoked pot when he was 19 several times. He was honest and admitted to all of this information in the police applications. Now at 26 he wants to apply to medical school. He has a clean record and good grades. Does his past affect him into getting into medical school? Does this information affect him into getting a DEA license number? Should he lie in the DEA application? Thank you

Pathdocmd

First and foremost, do NOT lie on the DEA application. Did he have any arrests and convictions? I do not know if that will hurt his chances of getting a DEA license number. That is a question better asked to an attorney that is familiar with such issues. About getting into med school... it will depend on the school. Some will say no and others may give a chance. I cannot say. If he lies and something comes up on a background check, then it is all over.


What kind of doctor should I be

I don't know what kind of Doctor to be. I want to be able to become a great mother and not be disconnected from my family; I would need hours that would allow me to do this. I also want to work in a hospital and directly help others (such as a surgeon). What are some kinds of doctors that could I become that would still allow me to have and be there for my children.

Pathdocmd

Almost all specialties have the direct contact with patients. I don't know what stage you are in pursuing medicine as a career, but if you are early I would not worry about it too much. You don't even have to know when you start medical school.


What can I do without a medical license

I was terminated from my residency program before I finished a year. I've tried time and time again to get back into a residency program or find a research position and I keep coming up with the, "we can't take you if you did not finish a full year". I'm wondering if there is anything I can do with my MD without a medical license?

casedentalmed

That's going to be challenging, but obviously you can't have hospital privileges. You can do something with your degree, but it would probably be helping with research or perhaps in medical journalism or advocacy. One website I found focuses on non-clinical jobs for physicians: http://www.nonclinicaljobs.com/p/search-jobs.html .


Hi, I'm a medical graduate from India, completed my 1yr intrnship, having keen interest in psychiatry. I just wanted to know, wheather The UK or The USA is best for my residency ? & what are the future job opportunities if I pursue my residency from the same!?

I wil b obliged, if you can enlighten me with the fellowship programs available! Fellowship in child psychiatry?

Pathdocmd

I can't say anything what the job opportunities are in the UK, but I think you need to decide if you want to like in the US or the UK.


I'm an IMG waiting for a US residency. I'm thinking of doing a Masters in Healthcare Administration in the Meantime. Any advice?

I'm an IMG waiting for a US residency. I'm thinking of doing a Masters in Healthcare Administration in the Meantime. Any advice?

I want a Masters that will serve as a plan B, with real career possibilities, in case the clinical residency doesn't work out for me but at the same time, I also want a degree that will compliment my MD in case I do go the clinical route. I know most people would probably consider a MPH first, but to be honest, I'm not really excited about that option. I'm seeking any and all advice out there. I know there are a lot of aspects I'm not aware of that might change my way of thinking. Please help.

tantacles

A masters will be of minimal help in acquiring a residency. However, it can very well serve as a plan B depending on what you wish to do. If you plan to do an MHA, I would suggest finding others who have tackled it first and see what their careers are like; this can help you determine whether you're a good fit for the degree.


For non-US IMGs, is there a way to pursue a less expensive PhD program? (the ones that grants and a small stipend)

I 've heard this happens only at Harvard or schools like that. Ivy Leagues only.

tantacles

Many schools that offer PhD's in the sciences offer a stipend and free tuition, including non-ivy league programs. Most programs will advertise this on their website. Notably, these programs may require a teaching fellowship in order to secure the funds.


How to get into medical school?

So I can't figure out a plan of action to take to allow myself to succeed getting accepted into a medical program. I applied this current cycle to maybe 20 MD/ DO programs and received a rejection letter from all of them. I also applied to three graduate (post bac or biomedical science) programs, and today received a rejection letter from one them. Im waiting to hear back from the other two, but I have a feeling I won't get into those either. This summer I plan on taking an accelerated EMT course and work as an EMT to boost my application this upcoming cycle 2016-2017. I will be retaking my MCAT May 14th as of now. I might delay it longer if I don't feel ready to take it. I've retaken most prerequisites for a better grade because my undergrad GPA was poor due to being a student athlete at a DI school. I've even taken extra science courses and done well in them. But now I feel like even if I receive a stellar MCAT score, med schools will not accept me because they will not think Im competitive enough for their programs. I'm at a loss and don't know what to do at this point anymore. What have others done that were successful in getting into medical school.

casedentalmed

As a D1 athlete, you understand the importance of strong coaching and a long-term plan. You also should know well what the standards for qualifying performance are in your sport and you should know what they are for medical education beyond just the numbers (though just like in sports, they are very important). If you haven't begun meeting with the health professions advisory team and talking to other premed students AND your evaluators, you should begin soon. Your professors should honestly give you feedback on your qualifications to get into medical school that has warranted the results of 20 rejections, or else it might be a matter of your timing or your relative lack of experience. It sounds like grades may be a big issue if you have been applying to postbac programs, and apparently having a suboptimal MCAT hasn't helped you.

You should also look at authentic resources like Aspiring Docs (https://students-residents.aamc.org/choosing-medical-career/medical-careers/aspiring-docs/) and AACOM (http://www.aacom.org/become-a-doctor). It sounds like you need to refocus your life and priorities, and that it might take longer than just a few months to correct. I hope your health professions advisors are honest in their feedback with you.


Med School in Bangalore, India vs. Undergrad + Med School in the U.S.

I am a high school senior who is currently thinking about going to Med School in India (M.S. Ramaiah Medical school ) because my family as a whole is moving over there (reason being, for my parents' work). I have applied to undergrad programs in the U.S.A and have gotten into UCLA, ASU Barrett's, and 8 other universities (Stony Brook, Hofstra, Stevens Institute of Technology,etc.). I applied for some 7-8 year accelerated med programs and did not get into any. I am having a hard time between deciding whether to either attend UCLA (great public university), go to Barrett's (it is in-state and would cost me very little to go there as I am a National Merit Finalist), or to go to college in India which seems to have more cons than pros (other than the supposed time saving). My plan is to finally practice as a doctor in India after med school, however, I would still like to do my residency in the US.

I have read many other Student Doctor threads about IMGs having a hard time getting matched in the US and I have also read a lot about how it will get worse in the future (less-nonexistent match rates ). I would like some advice on how I should approach these decisions which I have to make in a matter of 2-3 months. I understand the uphill battle I will face if I go to India for med school (lack of research opportunities, no USMLEs built into the curriculum), yet I am having a hard time weighing that against the possibility of not getting into med school in the US at all (many people do not get into med school right after undergrad).


casedentalmed

Only you can make the final decision on what you want to do. While you may be disappointed at not getting into an accelerated med program in the US, you cannot let the possibility of not getting into a US medical school sway your decisions so much if in the end you want to practice in the US. If your goal is to be a doctor in India, why spend so much more money in the US going to medical school? You'll essentially be in a similar accelerated program, but you would have to contend with serious culture shock and curricula that are probably more weed-out by design. And of course, you would have the great difficulty of practicing in the United States.

In general, only you have the power to really work hard in college and take advantage of the opportunities to mature and get a greater idea of the importance of medicine in society. Only you have the ability to do well in courses and your entrance exams to get into medical school (US or India or whatnot). It may be hard for you to understand at this time as a high school senior, and you may be looking for the quickest shortcuts to get you to where you want to go, but medicine is a hard profession requiring years of real dedication and passion even in the worst of times. Make sure you are not making decisions to do things as fast as possible, but make sure you are as well prepared with your academic, extracurricular, and personal foundation to be able to be successful for the next 5 decades as a professional.


Female surgeon

I want to do some form of surgery. I enjoy it and it interests me. I am a woman and I would like to have a family someday. I'm thinking of doing a general surgery residency. But surgeons are often on call and work long hours. Should I think of another field? What should I do? I know there are lots of female surgeons out there but...


casedentalmed

Why don't you talk to any female surgeons? I cannot discount that there is some chauvinism, but I would network with women who are already where you want to be. My first suggestion is the Association of Women Surgeons (https://www.womensurgeons.org/).

Of course, many women do become dentists and podiatrists; they perform surgeries, and those professions are considered among the most family-friendly in health care.


do we get soap offer without any IV calls

tantacles

It is unlikely. most programs in the SOAP will call to interview.


Is it possible to hold a full time IT Consultant job while going to med school?

tantacles

It is possible during the first two years, but it would be very difficult. Medical school requires the same or greater commitment as a full time job, and you will often be occupied for the 9-5 slot.

During 3rd year, it is absolutely impossible. Many rotations require upwards of 80 hours per week, and there just isn't time.

Fourth year? You could probably hold down a full time job during most of fourth year


Do I have a chance to get to medical school(M.D)?

Last year, my senior year, I had a family problem and I got one C in animal biology and one F in biochemistry II. Both were electives and not needed for my degree. I taught I would handle studying for MCAT and take 16 credit hour but the biochemistry II class I taught I would take and get a better grade in is taking too much of my time. So I am thinking of withdrawing form the class. How much would it affect my medical school acceptance? Should I even withdraw or just continue? What would you advise me? Do I even have a chance to get into medical school with F? I have a 3.82 GPA cumulative and 3.7 science GPA. I'm not looking to get into a top medical school, basically anywhere would do.

casedentalmed

It's too hard to say how much of an impact the F/W will have, but you need to be prepared to explain your situation. I think most people understand that things happen in life, but if you really think withdrawing from the course will help you cope with the situation, you should do it.


How to Find Quality Residencies

I am a current med student, and I am trying to make my list of residencies which I will apply to. Outside of geography, how can I make this decision? It's easy to figure out what the big names are in research and prestige, but what about other factors? I want a residency that has a good reputation for turning out quality doctors and is respected in the community. I've been looking through the FRIEDA and AAMC databases as well as the individual program websites, but the information they give is limited.

People tell you to find a residency that fits your personality, but how can I get a feeling for this short of going on an interview?

Also, how can I know if I have a reasonable shot at each of these residencies? I'm a middle-of-the-road USMD applicant looking to place in internal medicine, and I don't want to have a list that is too heavy in "reach" programs.

tantacles

I would strongly suggest that you consult your adviser about programs. If you don't have an adviser that can provide you with this information, doing an away rotation can get you access to more physicians that can potentially advise you.

Ultimately, there is no way to determine which programs will give you an interview, and only by going on interviews and consulting your peers and mentors will you be able to figure out which program is right for you.


Should I retake the MCAT?

I got a 514 but scored a 126 on Chem/Phys. My GPA is a 3.75 from a top 10 school. Should I retake to be competitive at a top medical school? I have my heart set on Hopkins.

I have research experience and all kinds of extracurricular/volunteer experience so is it worth it to aim for a 516?

casedentalmed

You should ask the admissions staff at the schools you are looking to apply to. It's too difficult to say that one subscore is going to undermine your application, but it all depends on how each admissions committee is going to handle it.


I want to know the best way to get prepared and confident that I'll ace the new MCAT.

Hello SND family,


I just finished my first semester at UMD, College Park with a 3.35 GPA. I've applied for the SMDEP sites for this upcoming summer, and I'm CNA/GNA certified. I'm a 19 year old African American male with dreams of becoming a Physician one day.

I want to get a head start on the new MCAT because I know a 3.8-4.0 GPA is unrealistic for me personally. So I hope with an excellent MCAT score, everything will balance out.

I'm taking BSCI105 this semester and Public Health is my major.


I've recent bought Kaplan Reading and Analysis, Princeton Math and Physics, and the Princeton Biology book.


My real question is: WHAT IS THE MOST EFFICIENT WAY, AS OF NOW, 2-3 YEARS IN ADVANCE,TO START PREPPING FOR THE NEW MCAT. I'll do anything.

I'm shooting for a 520-530 if possible.

tantacles

The best preparation for the MCAT in the long term is, ultimately, to do well in your coursework in the basic sciences and behavioral sciences. This also has the secondary effect of making you more attractive to medical schools. Remember that the MCAT also has a verbal portion, so performing critical analysis on texts in your coursework is also a good idea.

Ultimately, there are many different methods that you can use to prepare for the exam in the short term: You can study out of books, take a prep course, do practice exams, or do some combination of these. Just remember that it's important to take the MCAT and do your very best on your first try; the exam is expensive, and taking it a second time and doing better than your first attempt is reasonably uncommon. Therefore, I would recommend that you invest in and use practice exams to assess your potential performance.


Psychology Consult and Psychiatric evaluation in Residency

I got residency but I burnt out on my first day and went to ER to get help as I was unable to focus on my work and that seemed to be the best thing at that time. I am now going through psychiatric evaluation and psychology consults and I hope to get on board again. I asked the associate PD for putting me on vacation instead of medical leave and now I am in a confusion how do I join back and where to start again.

tantacles

Ultimately, the person to speak to is your program director. If you wish to continue in your program, your program director will be able to direct you to the appropriate resources, and if you wish to choose another specialty, you will need a letter from your program director in order to reapply.


46 year old EM MD wants to join military, does not need the retirement or benefits.

I am a 46 year old Emergency Med physician ABEM. I would like to join the military but do not want to complete a weekend a month. I do not need the benefits or retirement but I do want to continue working in Seattle as a civilian (kids, wife). I like the APMC program but do not know much about it. How many years am I obligated? My desire is to deploy once every 1-2 years to wherever needed. I have worked in Afghanistan and Iraq with NGO's so war is not a problem. Are there better options? Just trying to give back.....

Lee

First, thanks for your interest in serving the men and women of our armed forces! You can choose to serve in the Army Reserve, Navy Reserve or Air Force Reserve. You also could choose the Washington National Guard.

Being in Washington State, I'd recommend looking at the Army Reserve, potentially serving with the 396th Combat Support Hospital which is based in Vancouver, WA and Spokane, WA. You'll be able to drill with them during the summer and there is a good chance of being deployed every few years for a 90-day tour.

Call your local Reserve recruiter and ask for the local Reserve Healthcare Recruiter. You must speak with a healthcare recruiter -- they specialize in working with medical providers.


Becoming a doctor (anesthesiologist) and a marine

All my life I wanted to be a doctor. Something changed along the way, I want to be a marine. But I heard that marines don't have doctors they use the navy doctors. I read some information but it was a bit confusing and wasn't really relating to my question. I want train as a marine graduate as a marine, but I still have that desire to become a doctor. Is that still possible? I want to get my 20 years so I could retire.

Lee

You heard correctly, the Marines rely on the Navy for their medical care. If your desire is to be a Marine and a physician - those two things are mutually exclusive. But if you want to exclusively care for Marines and train with them as a Navy physician, that is something you can do.

First you'll need to join the Navy. There are a few different options such as Health Professions Scholarship Program (HPSP) and Uniformed Services University of the Health Sciences (USUHS). Then you'll need to serve as a "General Medical Officer" (GMO) or select a residency that fits with their needs (like Aerospace Medicine, anesthesia, general surgery). There are many opportunities to deploy with and serve tours with the Marines.

I'd recommend speaking with a Navy healthcare recruiter (not a regular recruiter) to get more options.


How will medical schools view this major?

I will be majoring in social work while taking pre-med prerequisites at one of the top schools. Will this major be beneficial for me as a physician and will medical schools view this major negatively or in any way at all?

tantacles

As a physician, the perspective of a social worker is invaluable. If you can weave this experience into your personal statement and make it a part of your narrative, it may have a positive impact on your application. However, ultimately, your major in and of itself will neither help nor harm you; all majors are acceptable for medical school.


Letters of Recommendation

Will my medical school application be considered without letters of recommendation from professors/ faculty but with multiple letters from practicing physicians?

casedentalmed

Every medical school will have its requirements on letters of evaluation on its website and the MSAR, and if you do not comply with their requirements, it's not a sure thing that your application will be seriously considered.


Did 10 months of Prelim IM year, now have an interview for Neurology PGY-2...will my incomplete preliminary year keep me from being offered a contract?

I competently completed 10 months of IM preliminary year, then resigned for a family emergency. Now I have been offered an interview at a PGY-2 Neurology program. They have reviewed all my documents and rotations completed.

Now I can't read the Program Director's mind, but I want to know if my 10 months will keep me from getting a contract. I am more than willing to do additional rotations to make up for the time. I want to know if I should bring this up in the interview or allow the PD to address it without my prompting.

Thanks!!

tantacles

It is hard to say whether you will not be offered this position, but it is likely that you will have to finish a PGY1 year to start work as a PGY-2. This may be something best brought up with the program director early so that you can rectify this deficiency.


I'm female, What should I wear while I shadow a trauma surgeon?

I am shadowing a trama surgeon this week. I am anxious, but nervous as well. What should I wear to look professional? (skirt? Pants? Blouse?) I would also appreciate it if you would be able to tell me what to expect.

casedentalmed

I would ask the trauma surgeon what would be appropriate.


I'm accepted to participate in clinical elective at the John Hopkins but I can not get feed back from registrars office

I'm accepted to participate in summer clinical elective John Hopkins by co director of orthopedic surgery. I've sent all my application papers. For 15 day, I have been waiting for the acceptance papers to prepare other requirements(visa, tickets..). I sent an email to orthopedic surgery department that I haven't got any email from them. They directed me to registrars office and notice that I have to fix my paper issues with them. According that, I sent an email to registrars office to ask for my acceptance letter and learn other requirements. The office directed me to the application page with an automatic message. http://www.hopkinsmedicine.org/som/VisMedStd/ApplctnPrcs.html I've read it several times when I was preparing papers to send. So, I couldn't get it, and asked for help again. The office didn't reply any of my emails. I asked the office that why am I still waiting for paper. No answer. Asked to orthopedic surgery department. No answer. Asked Professor's assistant. No answer. I don't know what to do. I'm kind of desperate, could you please help me with that?

tantacles

I would suggest that you send a formal e-mail to the office withdrawing from the elective so that you can choose a new one. If the office is not responding, it would be best to reschedule a different one.


Why wasn't I ranked?

I am new here, an IMG who just attempted the match for the second time. Received a total of 6 interviews.

My grades are not stellar but ok (220 step 1 and 225 step 2) I am an old grad. All i want is a family medicine residency. Well i did not match which did not surprise me since i canculated my chances at 43%. What surprised me was that one place i interviewed at, was in the SOAP which means they did not rank me at all nor did they rank the other guys ahead of me.

I do not think the interview went bad, i might have been nervous or awkward at worst, i did not say outrageous things, swore, or was rude to anybody. I was complimented on my English and CV.

Now, this is a rural program and i had to interview at different spots for the same program: the capital first and the small town rural programs next. Which one of them put me in the DNR list i will probably never know.

It seemed like i had extremely positive feedback from the capital and from more than one of those rural programs, one even promised to rank me first if i ranked them first but he obviously lied, the program that ended into SOAP was very positive to me but then did not rank me at all.

Looking back i really cannot find any justification to why i was not ranked, i am not a creeper, a freak or negatively stand out.

I am waiting for the SOAP hoping to get a second chance maybe they will re-think this since they have met me?

Or they are so disgusted they won't bother?

These programs only had 12 applicants, how many did they rank? This programs has both position unfilled.

Either there's something wrong with me or this is a strategy that i do not understand.

Why would program directors go as far as paying for hotels investing money and then do not rank an applicant who came all the way to your remote location? By the way i really enjoyed this program.

Next time i will lie about my first choice, had i told one of those PDs i ranked him first maybe I would be in?

tantacles

It is very difficult to say why you were not ranked by your program that you interviewed at. Residency program directors look at myriad factors when evaluating applicants. That being said, if you interviewed and were not ranked, there was likely a social or academic issue that contributed to your not being ranked. You mentioned you were an old graduate, for example - perhaps the residency program director did not wish to take anyone who had not graduated from medical school in the past four years. Perhaps you did not fit the program's mission, or perhaps they were simply looking for someone likely to stay in the area. It is truly impossible to say from an outside perspective.

I would suggest that if you plan to match again, you have a superior call these programs to find out what your issues are so that you can further improve them in the next cycle.


Medical Doctor Branching out to Biomedical Engineering

Many US schools offer a combined MD/Masters orPhD program where one studies for medical practice while specializing on biomedical engineering (Masters or Doctorate). Presumably those programs are for younger persons, non-medical doctors yet. But I haven't found a program for those who are already MDs.

I am an older guy. I know that I can tackle the science, the math and the practicum of this new discipline. Is there some experience or advice for me, or some other source I could investigate for information? Do I have to go back to their Medical School programs to study their biomedical engineering? Thank You and More Power To You!

Neuronix

I am an MD with PhD in biophysics, and I worked in a laboratory with students in both bioengineering and biophysics. I am unaware of any programs that you are looking for in the US. You could go back to school for a master's or doctorate in biomedical engineering, however it would be very difficult to balance this with continuing medical practice. The job market is much better in medical practice if you are board eligible or certified. Keep in mind that the grass is always greener, but medicine still has some of the greenest grass, especially for someone already in the field.

I would rather evaluate your career goals. Do you intend to do devices research? If so, your best bet may be to find a research-oriented fellowship at an academic medical center. In that kind of position you could continue seeing patients while getting research training.

If you want to discuss further, you can always PM me on the forums (Neuronix).


How much debt will I have when I graduate medical school?

tantacles

This largely depends on the total cost of the medical school you attend (taking into account scholarships and financial aid), how much debt you have from undergraduate, how much your family contributes to your medical education and expenses, and your ability to save while in medical school.

That being said, most private medical schools cost between $50,000-$70,000 to attend each year, so assuming you plan to pay this entire cost with loans, a safe estimate for the average student with no outstanding undergraduate loans and no parental financial contribution would be $200,000-$300,000 in debt after the final year of medical school.


What can I do as a physician besides see patients?

I realize the majority of physicians will be involved in patient care, but what else can I do as an MD?

casedentalmed

There are a number of things MD's do outside of the basic patient care responsibilities. There are a few profiles on the Aspiring Docs website at https://www.aamc.org/students/aspiring/career/ highlighting Kevin Pho, Andrew Ordon, David Parker, Norm Thagard, and Samuel Blacker.


What’s the best undergraduate major for medical school?

If I want to go to medical school, is there a specific major I should choose? I've heard that "major doesn't matter" but is that really true?

tantacles

Medical schools tend not to care about the major of their applicants; their main concern is that both your cumulative and science GPAs are both strong.

Knowing that, the best major for a medical student depends on two factors:

1. What major has classes that you will do well in? 2. What major are you interested in?

Try your best to get the best grades possible in every class and do well on your MCAT, but don't worry too much about your major. Ultimately, everyone studies the same material in medical school, so your undergraduate major has little bearing on your future in medicine.


How do I figure out what medical specialty I should choose?

How do I figure out which specialty is right for me? For the competitive specialties, I know I will need to start planning early and I am worried that if I decide later on a popular option like dermatology, I won't be able to get a residency.

smq123

You should always do the absolute best that you can in MS1, MS2, and MS3, so that, no matter what specialty you ultimately choose, you have a lot of options open to you. Get the highest Step 1 score that you can, do well on each exam, and get to know people in various departments. Shadow in different specialties - a day or two once in a while is fine - so that you have an idea of what being a dermatologist is like, or a radiologist, or an anesthesiologist.


How can I be successful in my clinical rotations?

I'm a second year medical student and I am really worried about clinical rotations. What can I do to make sure I don't look like an idiot on the wards?

Lee

Ask questions and be curious. If you don't have the answer, simply say "I don't know, but I will look it up." Read every night on the patients that you saw that day.

Be thorough and meticulous in everything that you do. With time you will become more efficient at doing patient interviews and writing notes.

Always arrive early, always expect to work late, and never ask to leave early -- when your preceptor or resident says it's time to go, then it's time to go. Some of your most valuable learning experiences come in the early morning and late evenings when it's just you and your preceptor or resident. Volunteer to take the hard cases, the weekend call, and the dirty work - it will pay dividends later in your career.

If you have any flexibility in your clinical rotations, always pick the hardest rotation sites - it will make you a better student, better resident, and practicing physician.